Onko Juha Himanka uskottava filosofi?

TiedeFilosofi

Filosofian dosentti Himanka hyökkää fenomenologisella asenteella fyysikoita Enqvist ja Valtaoja vastaan, kun he esimerkiksi väittävät Maan kiertävän Aurinkoa, tai että suhteellisuusteoria on totuudenmukainen kuvaus luonnosta.

Hyvä näin. Mutta kun lukee Himangan kritiikkiä, niin hänen ymmärtämättömyytensä fysiikasta aiheuttaa maatanielevän myötähäpeän tunteen! Miksi kukaan itseäänkunnioittava tieteentekijä lähtee tuollaiselle ikonoklastiselle harharetkelle, kauas omalta osaamisalueeltaan?

Himanka voi väittää, että plotemaiolainen, maakeskinen näkemys on toimiva, koska myös se selittää tähtitaivaan liikkeet. Mutta Himanka tekee itsestään täyden narrin, jos hän väittää maakeskisyyttä samanveroiseksi nykyfysiikan maailmankuvan kanssa, sillä esim. maakeskeisyys saisi jotkin taivaankappaleet liikkumaan oudosti, joskus pysähtymään ja kiertämään takaperin yms. Edelleen gravitaation vuoksi kaikki Aurinkokunnan planeetat kiertävät yhteistä massakeskipistettä, eikä se piste todellakaan sijaitse Maassa! Lisäksi Aurinkokunaan ulkopuolelta otetulla kuvamateriaalilla voi osoittaa kovapäisimmällekin, että planeetat kiertävät Auringon ympäri.

Himangan hyökkäys suhteellisuusteoriaa kohtaa on myös jotain typerryttävää! Kaksosparadoksia ei hänen mielestään voi selittää kiihtyvän liikkeen avulla, koska kiihtyvyyttä ei voi mitata, sillä "Einsteinin hississähän me nimenomaan emme tiedä olemmeko kiihtyvässä liikkeessä vai vetovoimakentässä. Vaaka ei tätä eroa kerro", mutta Himanka ei tunnu tajuavan, että avaruusalus EI ole Einsteinin hissi: esimerkiksi kiihdytyksessä astronautti painautuu tuoliaan vasten ja takuuvarmasti tuntee kiihtyvyyden.

http://ojs.tsv.fi/index.php/tt/article/view/3645/3415
http://ojs.tsv.fi/index.php/tt/article/view/8015/6098
http://ojs.tsv.fi/index.php/tt/article/view/2743/2515

Miten tuollainen pelle voi olla töissä yliopistolla? Sama jos joku toinen akateemisessa virassa oleva henkilö alkaisi väittämään, että Maa on litteä tai että Kuu on juustoa.

89

4169

    Vastaukset

    Anonyymi (Kirjaudu / Rekisteröidy)
    5000
    • Sehän on jonkinlainen luonnonlaki, että kokenut filosofi, saatuaan kylliksi julkaisuja alleen ja niistä itsevarmuutta, lähtee nolaamaan itsensä joko pohtimalla fysiikkaa tai sitten arvostelemalla fyysikkoja. ;)

      Ei se filosofin uskottavuutta filosofisissa kysymyksissä vähennä, vaikka hairahduksensa alalle, josta ei juuri mitään ymmärrä aiheuttaakin lukijassa myötähäpeän väristyksiä. Esimerkiksi Wittgenstein on edelleen suuri filosofi ja ajattelija, vaikka hänkin hairahtui oman suuruutensa sokaisemana kirjoittamaan huomautuksia väreistä.

      • filosoofi

        "Ei se filosofin uskottavuutta filosofisissa kysymyksissä vähennä"

        Kyllä muuten vähentää.


    • episyklomaniaa

      Lukaisin pikaisesti nuo kaikki Himangan tekstit ja minusta ne tarkkaan luettuna ja ymmärrettynä ovat varsin asiallisia. Sekä Enqvist että Valtaoja syyllistyvät usein populaareissa teoksissaan melko heikkoon filosofointiin mikä on samalla myös heikkoa tiedettä. Molemmille suosittelen keskittymistä lähinnä vain laskemiseen ja mielummin unohtamaan kokonaan kyökkifilosofoinnin.

      Ideologinen reduktionismi ei ole synonyymi tieteen ja tieteellisyyden kanssa vaikka niin usein kuvitellaan.

      ...

      "Himanka voi väittää, että plotemaiolainen, maakeskinen näkemys on toimiva, koska myös se selittää tähtitaivaan liikkeet. Mutta Himanka tekee itsestään täyden narrin, jos hän väittää maakeskisyyttä samanveroiseksi nykyfysiikan maailmankuvan kanssa, sillä esim. maakeskeisyys saisi jotkin taivaankappaleet liikkumaan oudosti, joskus pysähtymään ja kiertämään takaperin yms."

      Ptolemaiolainen episykliteoria todellakin toimii mutta sillä tavalla että jotkut planeetat näyttävät toisinaan vaihtavan suuntaa ja niiden radat muodostavat episyklejä. Episykliteoria kyllä ennustaa täysin pätevästi planeettojen liikkeet suhteessa maahan vaikka se onkin monimutkaisempi teoria kuin myöhäisempi Kopernikuksen aurinkokeskeinen malli.

      Occamin periaatteen kannalta siis aurinkokeskeinen teoria on parempi teoria kuin maakeskeinen koska se on paljon yksinkertaisempi - molemmat ovat kuitenkin yhteensopivia teorioita jos havainnot tehdään maakeskisesti.

      Kaikki onkin ok niin kauan kun oletetaan jonkinlaiset absoluuttiset koordinaatit avaruudessa tai jonkinlainen eetteri jonka suhteen taivaankappaleet liikkuvat.

      Ongelma syntyy Einsteinin suhteellisuusteorian myötä eli mitään absoluuttista liikettä jonkun eetterin suhteen ei voi enää olettaa. Kaikki liikkeet ovat suhteellisia toisiinsa nähden.

      " Lisäksi Aurinkokunaan ulkopuolelta otetulla kuvamateriaalilla voi osoittaa kovapäisimmällekin, että planeetat kiertävät Auringon ympäri."

      Sekin riippuu siitä miten se kuvaaja itse liikkuu suhteessa Maahan ja Aurinkoon.

      Suhtiksen perusteella sekä aurinkokeskeinen että maakeskeinen malli ovat molemmat virheellisiä.

      "Himanka ei tunnu tajuavan, että avaruusalus EI ole Einsteinin hissi: esimerkiksi kiihdytyksessä astronautti painautuu tuoliaan vasten ja takuuvarmasti tuntee kiihtyvyyden."

      Einsteinin hissin idea on että sieltä ei näe ulos eli ei voi tietää liikutaanko suhteessa ympäristöön. Voimakas gravitaatiokenttä aiheuttaa saman ilmiön kuin kiihtyvä liike eli niitä ei voi erottaa toisistaan.


      "Miten tuollainen pelle voi olla töissä yliopistolla? Sama jos joku toinen akateemisessa virassa oleva henkilö alkaisi väittämään, että Maa on litteä tai että Kuu on juustoa."


      Heh. Taitaa useimmille ihmisille filosofinen pohdiskelu olla yksinkertaisesti liian vaikeaa. Käytännön tasolla kyllä riittääkin common-sense mutuilu useimmissa tapauksissa.

      • Orwell-1984

        "Ptolemaiolainen episykliteoria" kun vain ei toimi!Miksi luulet Keplerin tapelleen Marsin radan kanssa jos Marsin liike olisi voitu täysin selittää episykliteorialla? Ja mitenkähän tuolla teorialla selitettäisiin planeettojen ratojen perihelin kiertymä? Entä miten episykliteorialla laskettaisiin avaruusalusten radat kun ne kiitävät avaruudessa pelkän painovoiman vaikuttaessa, siis käyttämättä moottoria (moottoria käytettäessä tatvittaisiin vielä Newtonin mekaniikkaa jota ei Ptolemaioksen aikana tunnettu). Jne,jne...


      • episyklomaniaa
        Orwell-1984 kirjoitti:

        "Ptolemaiolainen episykliteoria" kun vain ei toimi!Miksi luulet Keplerin tapelleen Marsin radan kanssa jos Marsin liike olisi voitu täysin selittää episykliteorialla? Ja mitenkähän tuolla teorialla selitettäisiin planeettojen ratojen perihelin kiertymä? Entä miten episykliteorialla laskettaisiin avaruusalusten radat kun ne kiitävät avaruudessa pelkän painovoiman vaikuttaessa, siis käyttämättä moottoria (moottoria käytettäessä tatvittaisiin vielä Newtonin mekaniikkaa jota ei Ptolemaioksen aikana tunnettu). Jne,jne...

        ""Ptolemaiolainen episykliteoria" kun vain ei toimi!"

        Se toimi astrologiassa mikä riitti hyvin sen ajan tarpeisiin.


        "Miksi luulet Keplerin tapelleen Marsin radan kanssa jos Marsin liike olisi voitu täysin selittää episykliteorialla?"

        Ei se Merkuriuksen ratakaan toimi Newtonin mukaisesti.

        Tieteessä on helppo olla jälkiviisas.

        Kuten jo edellä sanoin niin luin ne Himankan tekstit jo aikaisemmin eikä siellä pikaisella lukemisella löytynyt sellaisia älyttömyyksiä kuin mitä aloittaja esitti.

        Kehoitan sinuakin perehtymään varsinaiseen lähdetietoon ensin.


      • AvaiB.suora.vastaus
        episyklomaniaa kirjoitti:

        ""Ptolemaiolainen episykliteoria" kun vain ei toimi!"

        Se toimi astrologiassa mikä riitti hyvin sen ajan tarpeisiin.


        "Miksi luulet Keplerin tapelleen Marsin radan kanssa jos Marsin liike olisi voitu täysin selittää episykliteorialla?"

        Ei se Merkuriuksen ratakaan toimi Newtonin mukaisesti.

        Tieteessä on helppo olla jälkiviisas.

        Kuten jo edellä sanoin niin luin ne Himankan tekstit jo aikaisemmin eikä siellä pikaisella lukemisella löytynyt sellaisia älyttömyyksiä kuin mitä aloittaja esitti.

        Kehoitan sinuakin perehtymään varsinaiseen lähdetietoon ensin.

        Olekkonää se filosifimultinikki, joka aikoinaan kirjoitti, että jos premissit valitaan sopivasti, 1 1 = 3?

        A) Olen. B) En ole.


      • episyklomaniaa
        AvaiB.suora.vastaus kirjoitti:

        Olekkonää se filosifimultinikki, joka aikoinaan kirjoitti, että jos premissit valitaan sopivasti, 1 1 = 3?

        A) Olen. B) En ole.

        C) En kerro.... :-)


      • AvaiB
        episyklomaniaa kirjoitti:

        C) En kerro.... :-)

        Eipä ollut vaikea arvata.


      • episyklomaniaa
        AvaiB kirjoitti:

        Eipä ollut vaikea arvata.

        Ihan asiallisesti ottaen peruslogiikka toimii niin että jos saat mielivaltaisesti valita premissit loogisessa päättelyssä niin mikä tahansa on totta.

        Eli ts. jos yhteenlaskussa ei ole mitään ehtoja eikä siis premissejä niin

        1 1 on tänään 3, huomenna lentävä spagettihirviö ja ylihuomenna 666


      • AvaiB
        episyklomaniaa kirjoitti:

        Ihan asiallisesti ottaen peruslogiikka toimii niin että jos saat mielivaltaisesti valita premissit loogisessa päättelyssä niin mikä tahansa on totta.

        Eli ts. jos yhteenlaskussa ei ole mitään ehtoja eikä siis premissejä niin

        1 1 on tänään 3, huomenna lentävä spagettihirviö ja ylihuomenna 666

        Jos loogisessa järjestelmässä olisi ylimääräinen premissi, joka olisi jonkin muun premissin kanssa ristiriidassa, tässä järjestelmässä kaikki väitteet olisivat totta.

        Ts. järjestelmä olisi loogisen näköinen, mutta ei ansaitsisi nimitystä "looginen järjestelmä". Ei myöskään matemaattisen järjestelmän näköistä symbolipötköa, jossa vastakkaiset väitteet pitävät paikkansa, nimitetä "matemaattiseksi järjestelmäksi"

        Et tarkoittanut tätä alkuperäisessä lausumassasi, vaan kerroit, että jos premissit valittaisiin sopivasti, tämä olisi looginen järjestelmä, jossa 1 1 = 3. Sellaista ei ole. Jälkiviisastelu ylimääräisellä premissillä ei ole kunniksi. Selaa taaksepäin ja katso, mitä tulit sanoneeksi.

        Selittely, kiertely ja kaartelu, on nolompaa, kuin sen myöntäminen, että on erehtynyt. Tosin munauksesi on suuri, mutta sittenkin on helpompi irrottautua asiasta myöntämällä erehtyneensä, kuin jatkamalla selittelyä.


      • episyklomaniaa
        AvaiB kirjoitti:

        Jos loogisessa järjestelmässä olisi ylimääräinen premissi, joka olisi jonkin muun premissin kanssa ristiriidassa, tässä järjestelmässä kaikki väitteet olisivat totta.

        Ts. järjestelmä olisi loogisen näköinen, mutta ei ansaitsisi nimitystä "looginen järjestelmä". Ei myöskään matemaattisen järjestelmän näköistä symbolipötköa, jossa vastakkaiset väitteet pitävät paikkansa, nimitetä "matemaattiseksi järjestelmäksi"

        Et tarkoittanut tätä alkuperäisessä lausumassasi, vaan kerroit, että jos premissit valittaisiin sopivasti, tämä olisi looginen järjestelmä, jossa 1 1 = 3. Sellaista ei ole. Jälkiviisastelu ylimääräisellä premissillä ei ole kunniksi. Selaa taaksepäin ja katso, mitä tulit sanoneeksi.

        Selittely, kiertely ja kaartelu, on nolompaa, kuin sen myöntäminen, että on erehtynyt. Tosin munauksesi on suuri, mutta sittenkin on helpompi irrottautua asiasta myöntämällä erehtyneensä, kuin jatkamalla selittelyä.

        "Et tarkoittanut tätä alkuperäisessä lausumassasi, vaan kerroit, että jos premissit valittaisiin sopivasti, tämä olisi looginen järjestelmä, jossa 1 1 = 3."

        Tuo taitaa olla ikioma tulkintasi.

        Muistaakseni sanoin jotain sen tapaista että pitää määritellä mitä tarkoittaa " " merkki. 1 1=2 ei välttämättä pidä paikkaansa kaikissa mahdollisissa maailmoissa ilman mitään yhteenlaskun määritelmiä.

        Esim. hiukkanen ja antihiukkanen yhteentuotuna eivät ole 2 vaan lähinnä 0. Eliöiden lisääntymisessä 1 1 voi olla 3 tai enemmän jne. Taisin tuonkin silloin heittää lähinnä vitsinä mutta ei tällä foorumilla juuri kukaan ymmärrä huumoria.

        En jaksa edes muistaa niin vanhoja viestejä mutta mielestäni en ainakaan tarkoittanut mitään sellaista mitä väität minun tarkoittaneen. Jos sinulla on linkki siihen vanhaan alkuperäiseen viestiini niin mielelläni lukisin sen uudestaan ja kommentoin tarvittaessa. Sillä asialla ei kuitenkaan ole tämän ketjun aiheen kannalta yhtään mitään merkitystä. Laita siihen ketjuun kommenttisi jos on aihetta. Älä sabotoi tätä ketjua enempää....


      • AvaiB
        episyklomaniaa kirjoitti:

        "Et tarkoittanut tätä alkuperäisessä lausumassasi, vaan kerroit, että jos premissit valittaisiin sopivasti, tämä olisi looginen järjestelmä, jossa 1 1 = 3."

        Tuo taitaa olla ikioma tulkintasi.

        Muistaakseni sanoin jotain sen tapaista että pitää määritellä mitä tarkoittaa " " merkki. 1 1=2 ei välttämättä pidä paikkaansa kaikissa mahdollisissa maailmoissa ilman mitään yhteenlaskun määritelmiä.

        Esim. hiukkanen ja antihiukkanen yhteentuotuna eivät ole 2 vaan lähinnä 0. Eliöiden lisääntymisessä 1 1 voi olla 3 tai enemmän jne. Taisin tuonkin silloin heittää lähinnä vitsinä mutta ei tällä foorumilla juuri kukaan ymmärrä huumoria.

        En jaksa edes muistaa niin vanhoja viestejä mutta mielestäni en ainakaan tarkoittanut mitään sellaista mitä väität minun tarkoittaneen. Jos sinulla on linkki siihen vanhaan alkuperäiseen viestiini niin mielelläni lukisin sen uudestaan ja kommentoin tarvittaessa. Sillä asialla ei kuitenkaan ole tämän ketjun aiheen kannalta yhtään mitään merkitystä. Laita siihen ketjuun kommenttisi jos on aihetta. Älä sabotoi tätä ketjua enempää....

        "Muistaakseni sanoin jotain sen tapaista että pitää määritellä mitä tarkoittaa " " merkki. 1 1=2 ei välttämättä pidä paikkaansa kaikissa mahdollisissa maailmoissa ilman mitään yhteenlaskun määritelmiä."

        Muistisi on sinulle hyvin armollinen.

        Sanoit, että jos premissit valitaan sopivasti, 1 1 = 3. Siihen vastasin, että sehän on mainio keino rikastua. Kun olet minulle velkaa euron ja minulla on ennestään euro, ja maksat velkasi, minulla ei olekaan kaksi euroa vaan miljoona euroa, sillä olen valinnut premissit näin. Keskustelu päättyi kuin seinään.

        Kuvitelma, että yhteenlaskun säännöt voisivat muuttua, on vähän samanlainen, kuin väite, että rautanaula liukenee Coca Cola -lasiin yhdessä yössä. Aikoinaan se tuntui niin mielenkiintoiselta, että se pantiin eteenpäin, ennenkuin kukaan kokeili.

        Yhteenlaskun muuttuminen on filosofisissa ja uskonnollisissa piireissä samanlailla kulkeva huhu, joka pannaan eteenpäin, koska se on niin mielenkiintoinen, ei siksi, että se olisi tosi. Eräs teologi väitti huhua todeksi vielä senkin jälkeen, kun olin sanonut, ettei se pidä paikkaansa.

        Mieti vähän lisääkö vai vähentääkö kiertely uskottavuuttasi.


      • episyklomaniaa
        AvaiB kirjoitti:

        "Muistaakseni sanoin jotain sen tapaista että pitää määritellä mitä tarkoittaa " " merkki. 1 1=2 ei välttämättä pidä paikkaansa kaikissa mahdollisissa maailmoissa ilman mitään yhteenlaskun määritelmiä."

        Muistisi on sinulle hyvin armollinen.

        Sanoit, että jos premissit valitaan sopivasti, 1 1 = 3. Siihen vastasin, että sehän on mainio keino rikastua. Kun olet minulle velkaa euron ja minulla on ennestään euro, ja maksat velkasi, minulla ei olekaan kaksi euroa vaan miljoona euroa, sillä olen valinnut premissit näin. Keskustelu päättyi kuin seinään.

        Kuvitelma, että yhteenlaskun säännöt voisivat muuttua, on vähän samanlainen, kuin väite, että rautanaula liukenee Coca Cola -lasiin yhdessä yössä. Aikoinaan se tuntui niin mielenkiintoiselta, että se pantiin eteenpäin, ennenkuin kukaan kokeili.

        Yhteenlaskun muuttuminen on filosofisissa ja uskonnollisissa piireissä samanlailla kulkeva huhu, joka pannaan eteenpäin, koska se on niin mielenkiintoinen, ei siksi, että se olisi tosi. Eräs teologi väitti huhua todeksi vielä senkin jälkeen, kun olin sanonut, ettei se pidä paikkaansa.

        Mieti vähän lisääkö vai vähentääkö kiertely uskottavuuttasi.

        Varmaat olet sitten keskustellut jonkun muun kanssa kun en muista noita kommenttejasi. Omassa vanhassa viestissäni lähinnä esitin lyhyesti myös että yhteenlaskettavien objektien pitää olla riittävän erillisiä jotta niitä voisi edes mielekkäästi laskea yhteen. Yhteenlasku ei ylipäätänsä ole sovellettavissa kaikkiin mahdollisiin konkreettisiin tilanteisiin.

        Abstraktissa matematiikassa yhteenlasku on määritelty tarkasti:

        https://fi.wikipedia.org/wiki/Yhteenlasku

        Kun yhteenlaskun premissit on näin määritelty niin 1 1=2 aina. Jos yhteenlaskun premissit ovat puutteellisesti tai ei ollenkaan määriteltyjä niin kyseinen symbolijono voi tarkoittaa mitä tahansa.

        Ei se yhteenlasku ole mikään ikuinen totuus vaan lähinnä ihmisten välinen konstruktio ja sopimus ja se yhteenlasku soveltuu parhaiten konkreettisten esineiden kaupankäyntiin, ruokaresepeteihin yms.

        ....


        Tuolta voit etsiä lopuksi oman mokasi tässä keskustelussa. Toivottavasti opit jotain.

        https://fi.wikipedia.org/wiki/Argumentum_ad_hominem (kaivon myrkyttäminen)

        En enempää viitsi kommentoida sinun harhaisiin sekoiluihisi....


      • AvaiB
        episyklomaniaa kirjoitti:

        Varmaat olet sitten keskustellut jonkun muun kanssa kun en muista noita kommenttejasi. Omassa vanhassa viestissäni lähinnä esitin lyhyesti myös että yhteenlaskettavien objektien pitää olla riittävän erillisiä jotta niitä voisi edes mielekkäästi laskea yhteen. Yhteenlasku ei ylipäätänsä ole sovellettavissa kaikkiin mahdollisiin konkreettisiin tilanteisiin.

        Abstraktissa matematiikassa yhteenlasku on määritelty tarkasti:

        https://fi.wikipedia.org/wiki/Yhteenlasku

        Kun yhteenlaskun premissit on näin määritelty niin 1 1=2 aina. Jos yhteenlaskun premissit ovat puutteellisesti tai ei ollenkaan määriteltyjä niin kyseinen symbolijono voi tarkoittaa mitä tahansa.

        Ei se yhteenlasku ole mikään ikuinen totuus vaan lähinnä ihmisten välinen konstruktio ja sopimus ja se yhteenlasku soveltuu parhaiten konkreettisten esineiden kaupankäyntiin, ruokaresepeteihin yms.

        ....


        Tuolta voit etsiä lopuksi oman mokasi tässä keskustelussa. Toivottavasti opit jotain.

        https://fi.wikipedia.org/wiki/Argumentum_ad_hominem (kaivon myrkyttäminen)

        En enempää viitsi kommentoida sinun harhaisiin sekoiluihisi....

        Jatkat näköjään samalla linjalla: "Ei se yhteenlasku ole mikään ikuinen totuus vaan lähinnä ihmisten välinen konstruktio ja sopimus..." Että sillä tavalla.


      • episyklomaniaa
        AvaiB kirjoitti:

        Jatkat näköjään samalla linjalla: "Ei se yhteenlasku ole mikään ikuinen totuus vaan lähinnä ihmisten välinen konstruktio ja sopimus..." Että sillä tavalla.

        Sinulla on ilmeisesti vähän erilainen käsitys matematiikasta kuin minulla. Itse asiassa matematiikan filosofia käsittelee juuri näitä asioita:

        https://fi.wikipedia.org/wiki/Matematiikan_filosofia

        http://plato.stanford.edu/entries/philosophy-mathematics/


        Siellä on useita eri vaihtoehtoja joista varmaan löydät myös oman kantasi. Aiheesta voit sitten halutessasi aloittaa ketjun. Sinun omaksumasi kuvitelma matematiikasta ei todellakaan ole ainoa mahdollinen.

        Tämä on taas edelleen rankasti aiheen vierestä joten lopeta tuo jankuttaminen jo.


      • AvaiB
        episyklomaniaa kirjoitti:

        Sinulla on ilmeisesti vähän erilainen käsitys matematiikasta kuin minulla. Itse asiassa matematiikan filosofia käsittelee juuri näitä asioita:

        https://fi.wikipedia.org/wiki/Matematiikan_filosofia

        http://plato.stanford.edu/entries/philosophy-mathematics/


        Siellä on useita eri vaihtoehtoja joista varmaan löydät myös oman kantasi. Aiheesta voit sitten halutessasi aloittaa ketjun. Sinun omaksumasi kuvitelma matematiikasta ei todellakaan ole ainoa mahdollinen.

        Tämä on taas edelleen rankasti aiheen vierestä joten lopeta tuo jankuttaminen jo.

        Matematiikka ei ole millään muotoa filosofinen kysymys, vaikka filosofit tyrkyttävät omia kuvitelmiaan.


      • episyklomaniaa
        AvaiB kirjoitti:

        Matematiikka ei ole millään muotoa filosofinen kysymys, vaikka filosofit tyrkyttävät omia kuvitelmiaan.

        "Matematiikka ei ole millään muotoa filosofinen kysymys, vaikka filosofit tyrkyttävät omia kuvitelmiaan."

        Filosofinen kantasi on huomioitu.


    • Eipä tietenkään ole uskottava filosofi, mutta filosofeihinhan menee fenomenologiset puheet kuin väärä raha.

      Opin perusideahan on, että tietoisuus on jotakin erilaista kuin hiukkasten liike. Tarvitaan siis erityinen tietoteoria selittämään, miten maailman ilmiöt saavuttavat sisimmän ajattelun. Tämä sitten poikii vaikka minkä verran pohdintaa herättäviä lisäkysymyksiä, kuten kuvitelman vahvasta emergenssistä.

      Hermeneutiikkaahan on tarjottu Helsingin yliopistossa jo vuosikymmeniä. Kyllä sekin perinne joskus katkeaa, mutta tilalle tulee vain yhtä sekavaa stiignafuuliaa.

      Jos hermeneutikot olisivat alun pitäen omistaneet korkemman intelligenssin, heistä olisi mahdollisesti tullut luonnontieteilijöitä. Sekä geenit että kasvatus ovat kuitenkin vieneet heidät vikaraiteelle, josta paluuta ei ole. Suuri ongelma on tietysti, että niin hermeneutikot kuin muutkin uskonnollismieliset ovat varustetut rautaisella itseluottamuksella, ja rupeavat lopulta kritisoimaan tieteellistä työskentelyä. Siinähän sitä sitten ollaan.

      Prujuahan voi työntää loputtomiin, kun muistaa sanoa, että tämä on sanottu siinä ja siinä kontekstissa, reunaehdoin tai teoriaa noudattaen. Humanistisessa tutkimuksessa kaikki käy, eri asia on sitten, mitä arvoa tällä on, mutta alan opiskelijoihin voi tehdä hyvin syvällisen vaikutuksen.

      • episyklomaniaa

        "Opin perusideahan on, että tietoisuus on jotakin erilaista kuin hiukkasten liike. "

        Ei ole. "Hiukkasten" liike on tietoisuudessa kuten kaikki muukin koettu.

        "Tarvitaan siis erityinen tietoteoria selittämään, miten maailman ilmiöt saavuttavat sisimmän ajattelun. "


        Ei tarvita. Kaikki mahdollinen kokemus aistikokemukset mukaanluettuna ovat tietoisuudessa. Materialismissa tietoisuus pitää johtaa hiukkasten liikkeistä koska oletetaan reduktionistisesti että kaikki koostuu hiukkasista ja niiden liikkeestä ajassa ja avaruudessa. Ns aine on se osa tietoisuuden sisältöä joka on säännönmukaista ja kaikille yksilöllisille tietoisuuksille yhteistä.

        Fenomenologia on filosofisesti paljon yksinkertaisempi kuin mikä tahansa materialistinen malli tai kuvaus.

        Materialismi ja varsinkin reduktionistinen mekanismi on ollut henkitoreissaan jo yli sata vuotta kvanttifysiikan ja suhteellisuusteorian syntymisen myötä. Ei ole olemassa mitään kiinteää ainetta joka olisi lokalisoitavissa ajallisesti ja paikallisesti johonkin ja josta kaikki muu jotenkin muka koostuisi. Fysiikassakin puhutaan kentistä ja ne ns. hiukkaset ovat lähinnä sen kentän eksitaatiota. Kenttäkäsite sinänsä on non-lokaali eli se vaikuttaa joka paikassa jos sitä 'paikka´ - käsitettä ylipäätänsä on enää mielekästä käyttää.


        Viimeisen naulan ideologisen reduktionismin eli modernin materialismin arkkuun naulasi delayed-choise-experiment koe joka osoittaa kiistatta että lokaali realismi ei voi pitää paikkaansa.

        Mikään ei tarkkaan ottaen liiku mihinkään eikä mitään pienten biljardipallojen kaltaisia aineen perusyksiköitä eli hiukkasia ole olemassakaan. Oikeastaan fysiikassa voidaan ilmaista asioita vain matemaattisesti - ymmärrettävissä oleva kuvaus ei ole mahdollinen arkikielen ja arkiymmärryksen puitteissa. "Shut up and calculate" on hyvä neuvoa Enqvistille, Valtaojalle ja lukemattomille muille heidän kaltaisilleen. Nykyfysiikka ei kerta kaikkiaan ole kuvattavissa kadunmiehen common-sense terminologialla ilman filosofista tahatonta huumoria tai muuta sekoilua.

        Sekä Enqvistin että Valtaojan populistiset kirjoitukset ovat lähinnä naiivia lokaalia realismia joka uppoaa ns. "kuluttajiin" eli aivopestyyn hömppäkansaan kuin häkä....


        Ainoa varma pohja tieteelle ja filosofialle on lähteä rakentamaan maailmankuvaa sellaisesta mikä on ehdottoman varmaa ja kiistatonta - nimittäin tietoisuus ja sen sisällöt.

        ....

        Niin Al Jabr olemme keskustelleet tästä usein ennenkin.... :-)

        Hyvää joulua ja uuttavuotta sinullekin!


      • VivaFilosofia

        Eihän se ressukka Himanka edes tietäisi, mistä Aurinko tai tähdet koostuvat, ellei joku luonnontieteilijä sitä hänelle olisi kertonut! Mutta ehkä Himankalla on skitsohäiriö, ja hän haluaa selittää kaiken siten, kuten se hänelle itselleen sopii. Ei ihmiskunta tarvitse objektiivista tiedettä, eläköön kivikausi!

        Eli miksi puhua alkuaineista tai luonnon perusvoimista, jos jokainen voi jäsentää maailman, miten haluaa - ja olla yhtä oikeassa kuin muutkin.


      • episyklomaniaa

        Sen verran voisin puolustaa Enqvistiä ja Valtaojaa Himakan kritiikkiä vastaan että
        kyse on kansantajuistamisesta eli he kirjoittavat kirjojaan tavalliselle suomalaiselle keskiverto ihmiselle joka puurtaa työssään 8t päivässä eikä jaksa töiden jälkeen kovin paljon muuhun keskittyä kuin telkkarin katsomiseen ja keskikaljan kittaamiseen.

        Tavallisen ihmisen keskittymiskyky ei nykyään paljoa ylitä 3 minuuttia joten kirjoja vielä lukevat lienevät lähinnä kansan älyllistä eliittiä ja hekin lienevät korkeammalla tasolla kuin suurin osa s24 tiedepalstalle kirjoittavista henkilöistä.

        Enqvist tuskin on tarkoittanut että hänen kirjoituksiaan arvostellaan tiukan akateemisen filosofisen analyysin pohjalta eikä hän myöskään varmaan ole osoittanut teoksiaan omille vertaisilleen eli muille fyysikoille.

        Tehdäkseen tiedettä kansantajuiseksi on tehtävä kompromisseja ymmärrettävyyden ja tarkkuuden välillä. Kansantajuisissa tieteellisissä teksteissä on usein ns. "valkoisia valheita" eli niissä esitetyt väitteet eivät välttämättä pidä paikkaansa ihan tarkasti. Lukijalla ei siis edellytetä olevan perustietoja fysiikasta.

        Filosofia ei ole mikään luonnontieteiden vastakohta vaan molemmat tarvitsevat toisiaan ainakin perustutkimuksen tasolla. Kaikki tieteet ovat aikoinaan irrottautuneet filosofiasta jota aikaisemmin kutsuttiin luonnonfilosofiaksi. Fysiikkaan liittyvät filosofiset pohdinnat tapahtuvat nykyään ja viimeiset 100 vuotta lähinnä fysiikkatieteen sisällä ja huippufyysikoiden toimesta.


        Himankaa on helppo kritisoida vain sillä perusteella että hän on filosofi mutta veikkaan että tuskin kovin moni edes on vaivautunut lukemaan alussa linkitettyjä pdf-tiedostoja ajatuksella niin että kritiikissä olisi jotain pohjaa olemassa (Nimimerkki al-jabr ei ainakaan koska hän reagoi aina sanaan "filosofia" samalla tavalla kuin härkä punaiseen vaatteeseen - älyllisen pohdinnan ollessa siten lähinnä Pavlovin koiran tasoa)


      • Anonyymi
        episyklomaniaa kirjoitti:

        "Opin perusideahan on, että tietoisuus on jotakin erilaista kuin hiukkasten liike. "

        Ei ole. "Hiukkasten" liike on tietoisuudessa kuten kaikki muukin koettu.

        "Tarvitaan siis erityinen tietoteoria selittämään, miten maailman ilmiöt saavuttavat sisimmän ajattelun. "


        Ei tarvita. Kaikki mahdollinen kokemus aistikokemukset mukaanluettuna ovat tietoisuudessa. Materialismissa tietoisuus pitää johtaa hiukkasten liikkeistä koska oletetaan reduktionistisesti että kaikki koostuu hiukkasista ja niiden liikkeestä ajassa ja avaruudessa. Ns aine on se osa tietoisuuden sisältöä joka on säännönmukaista ja kaikille yksilöllisille tietoisuuksille yhteistä.

        Fenomenologia on filosofisesti paljon yksinkertaisempi kuin mikä tahansa materialistinen malli tai kuvaus.

        Materialismi ja varsinkin reduktionistinen mekanismi on ollut henkitoreissaan jo yli sata vuotta kvanttifysiikan ja suhteellisuusteorian syntymisen myötä. Ei ole olemassa mitään kiinteää ainetta joka olisi lokalisoitavissa ajallisesti ja paikallisesti johonkin ja josta kaikki muu jotenkin muka koostuisi. Fysiikassakin puhutaan kentistä ja ne ns. hiukkaset ovat lähinnä sen kentän eksitaatiota. Kenttäkäsite sinänsä on non-lokaali eli se vaikuttaa joka paikassa jos sitä 'paikka´ - käsitettä ylipäätänsä on enää mielekästä käyttää.


        Viimeisen naulan ideologisen reduktionismin eli modernin materialismin arkkuun naulasi delayed-choise-experiment koe joka osoittaa kiistatta että lokaali realismi ei voi pitää paikkaansa.

        Mikään ei tarkkaan ottaen liiku mihinkään eikä mitään pienten biljardipallojen kaltaisia aineen perusyksiköitä eli hiukkasia ole olemassakaan. Oikeastaan fysiikassa voidaan ilmaista asioita vain matemaattisesti - ymmärrettävissä oleva kuvaus ei ole mahdollinen arkikielen ja arkiymmärryksen puitteissa. "Shut up and calculate" on hyvä neuvoa Enqvistille, Valtaojalle ja lukemattomille muille heidän kaltaisilleen. Nykyfysiikka ei kerta kaikkiaan ole kuvattavissa kadunmiehen common-sense terminologialla ilman filosofista tahatonta huumoria tai muuta sekoilua.

        Sekä Enqvistin että Valtaojan populistiset kirjoitukset ovat lähinnä naiivia lokaalia realismia joka uppoaa ns. "kuluttajiin" eli aivopestyyn hömppäkansaan kuin häkä....


        Ainoa varma pohja tieteelle ja filosofialle on lähteä rakentamaan maailmankuvaa sellaisesta mikä on ehdottoman varmaa ja kiistatonta - nimittäin tietoisuus ja sen sisällöt.

        ....

        Niin Al Jabr olemme keskustelleet tästä usein ennenkin.... :-)

        Hyvää joulua ja uuttavuotta sinullekin!

        "Materialismi ja varsinkin reduktionistinen mekanismi on ollut henkitoreissaan jo yli sata vuotta kvanttifysiikan ja suhteellisuusteorian syntymisen myötä. Ei ole olemassa mitään kiinteää ainetta joka olisi lokalisoitavissa ajallisesti ja paikallisesti johonkin ja josta kaikki muu jotenkin muka koostuisi. Fysiikassakin puhutaan kentistä ja ne ns. hiukkaset ovat lähinnä sen kentän eksitaatiota. Kenttäkäsite sinänsä on non-lokaali eli se vaikuttaa joka paikassa jos sitä 'paikka´ - käsitettä ylipäätänsä on enää mielekästä käyttää."

        Kenttäkäsite on aina lokaali. Fysiikassa on kiinnostuttu siitä, muuttuuko kenttä lokaalisten vain ei-lokaalisten sääntöjen mukaan. Kentän itse pitää olla jokin luku tai matemaattinen objekti, ja pitää olla avaruus, jonka jokaisessa paikassa on yksi näistä objekteista. Lähes kaikki mitä tällaisesta järjestelystä voidaan sanoa, on lokaalisti sanottua yhdestä paikasta ja sen objektista kerrallaan. Avaruus itse on jo muodostettu käyttäen reduktionismia eli on ajateltu, että siinä voidaan tarkastella äärettömän pieniä osasia ja etäisyyksiä. Sanoessa, että avaruudessa on jokin kenttä, tulee jatkaneeksi tätä reduktionismia eli puhuu objektista joka jakautuu infinitesimaalisiin osiin (joilla on kaikki merkitys, jos kentän muuttumisesta sanoo jotain matemaattista). Jos matemaattisia objekteja sanoo olevan jossain muualla kuin aika-avaruudessa, niitä ei tarvitse kutsua enää kentiksi, mutta aika moni matemaattinen objekti on redusoitu tai redusoiva tai mitä hyvänsä.

        Ei-lokaalisti-muuttuva kenttä voi olla esim. kahden toisistaan avaruudessa erotetun eksitaation välillä tehty operaatio, joka muokkaa eksitaation rakennetta täysin tai kaikista sen pienemmistä kohdista kerralla ilman, että kohdat vaikuttavat yhden eksitaation sisällä toisiinsa (Jos eksitaatio edes on paikka-avaruudessa aaltoileva silloinkin tätä tehdään myös lokaalisti ja tarkoitetaan kahden objektin todennäköisyyden peittävän toisensa paikassa, mutta tapahtumatodennäköisyyden olevan sitä että vain täysi lokalisoituminen tuottaa sitä oli se missä hyvänsä). Yleensä tätä pidettäisiin hiukkasluontoisena tapahtumana tai toimenpiteenä kentälle, koska ainakaan aallot (matemaattiset objektit ja yhtälöt) eivät käyttäydy näin (näistä kahdesta aallot ovat inifinitesimaalisesti reduktionistisia, ja hiukkaset ovat reduktionistisia vain muulla tavalla eli lähinnä verrattuna makromaailmaan). Vertaa hiukkasluonteen määritelmää eksitaatioiden käsittelyssä myös siihen, että on olemassa QM:n aaltofunktio, jonka sanotaan olevan kuin hiukkanen niissä tapauksissa, joissa matemaattiseen objektiin nimeltä aalto vaikutetaan ja se romahtaa (eksitaatio itse on aalto, mutta epä-lokaali tai lokaali operaatio siihen ei ole aina aaltomekaaninen, mikä olisi aina lokaalia). Eksitaatioita siellä ja täällä on tietysti parempi nimitys asialle.

        QFT ei ole vain kentän eksitaatio, vaan siinä on toisistaan täysin erotetttavien eksitaatioiden olemista ja ei-olemista kuvaava matemaattinen objekti nimeltä operaattori. ja jokaisen sellaisen vaikuttamista yhdessä avaruuden pisteessä (joko vakuumiin tai siihen mitä siellä on) pidetään myös sinä että, jokaisessa avaruuden pisteessä on joukko operaattoreita eli matemaattinen objekti eli kenttä, joka onkin Q-kenttä. Tällöin asia kuten vetovoima tai hylkimisvoima voi saada hiukkasluonteen virtuaalihiukkasina, mutta sekin tarkoittaa eksitaatioita. Vuorovaikutus pitää jakaa näihin eksitaatioihin, joita on jokin laskettava kokonaisluku-määrä aivan kuten hiekan hiukkasia pullossa, ja tässä on paljon kreikkalaisen reduktionismin (atomismin) piirteitä eikä vain infinitesimaalisen reduktionismin, kuten 1700-1800-luvulla oltiin totuttu.

        Infinitesimaalisessa reduktionismissa ei ole mitään kiinteää, jos tarkoitetaan ettei olisi mitään jakamatonta. Jos on olemassa materiaalifyysikkoja, he osaavat infinitesimaalisin menetelmin kertoa vielä tarkemman määritelmän sille, mikä makrokappaleista tulee olemaan kiinteä ja mikä ei. QFT:ssä kvantit voivat olla jakamattomia ja kiinteitä. Varsinkin kun niiden energian tai muiden suureiden spektri on epäjatkuva ja nimenomaan pistemäinen. Kuten kenttäeksitaatioiden tapauksessa oletetaan aina, jos niillä on sisäinen avaruus, jossa esiintyy aina potentiaali, ja jos se aiheuttaa vain kvanttioskillaatio-ratkaisuja. Kvantissa itsessään esiintyy todennäköisesti jotain infinitesimaalisia matemaattisia objekteja.

        "Kenttäkäsite sinänsä on non-lokaali eli se vaikuttaa joka paikassa "

        Jo klassisissa kentissä kentän pitää vaikuttaa itseensä. Tämä vaikutus on ajateltu vain ajattelemalla vielä lyhyempiä etäisyyksiä ja aikavälejä. Kun nämä välit vaikuttavat vain toisiinsa puhutaan selvästi lokaalisuudesta siinä, miten kenttä muuttuu.

        Toinen tapa ymmärtää kentän vaikuttaminen, on sanoa, että kenttä vaikuttaa vain jos on olemassa toinen objekti. Mikä vaikuttaa kenttään? Mihin kenttä vaikuttaa? Ja millä perusteella kenttä ja objektit kokisivat vaikutuksen mistä ja mihin tahansa?

        1


      • Anonyymi
        Anonyymi kirjoitti:

        "Materialismi ja varsinkin reduktionistinen mekanismi on ollut henkitoreissaan jo yli sata vuotta kvanttifysiikan ja suhteellisuusteorian syntymisen myötä. Ei ole olemassa mitään kiinteää ainetta joka olisi lokalisoitavissa ajallisesti ja paikallisesti johonkin ja josta kaikki muu jotenkin muka koostuisi. Fysiikassakin puhutaan kentistä ja ne ns. hiukkaset ovat lähinnä sen kentän eksitaatiota. Kenttäkäsite sinänsä on non-lokaali eli se vaikuttaa joka paikassa jos sitä 'paikka´ - käsitettä ylipäätänsä on enää mielekästä käyttää."

        Kenttäkäsite on aina lokaali. Fysiikassa on kiinnostuttu siitä, muuttuuko kenttä lokaalisten vain ei-lokaalisten sääntöjen mukaan. Kentän itse pitää olla jokin luku tai matemaattinen objekti, ja pitää olla avaruus, jonka jokaisessa paikassa on yksi näistä objekteista. Lähes kaikki mitä tällaisesta järjestelystä voidaan sanoa, on lokaalisti sanottua yhdestä paikasta ja sen objektista kerrallaan. Avaruus itse on jo muodostettu käyttäen reduktionismia eli on ajateltu, että siinä voidaan tarkastella äärettömän pieniä osasia ja etäisyyksiä. Sanoessa, että avaruudessa on jokin kenttä, tulee jatkaneeksi tätä reduktionismia eli puhuu objektista joka jakautuu infinitesimaalisiin osiin (joilla on kaikki merkitys, jos kentän muuttumisesta sanoo jotain matemaattista). Jos matemaattisia objekteja sanoo olevan jossain muualla kuin aika-avaruudessa, niitä ei tarvitse kutsua enää kentiksi, mutta aika moni matemaattinen objekti on redusoitu tai redusoiva tai mitä hyvänsä.

        Ei-lokaalisti-muuttuva kenttä voi olla esim. kahden toisistaan avaruudessa erotetun eksitaation välillä tehty operaatio, joka muokkaa eksitaation rakennetta täysin tai kaikista sen pienemmistä kohdista kerralla ilman, että kohdat vaikuttavat yhden eksitaation sisällä toisiinsa (Jos eksitaatio edes on paikka-avaruudessa aaltoileva silloinkin tätä tehdään myös lokaalisti ja tarkoitetaan kahden objektin todennäköisyyden peittävän toisensa paikassa, mutta tapahtumatodennäköisyyden olevan sitä että vain täysi lokalisoituminen tuottaa sitä oli se missä hyvänsä). Yleensä tätä pidettäisiin hiukkasluontoisena tapahtumana tai toimenpiteenä kentälle, koska ainakaan aallot (matemaattiset objektit ja yhtälöt) eivät käyttäydy näin (näistä kahdesta aallot ovat inifinitesimaalisesti reduktionistisia, ja hiukkaset ovat reduktionistisia vain muulla tavalla eli lähinnä verrattuna makromaailmaan). Vertaa hiukkasluonteen määritelmää eksitaatioiden käsittelyssä myös siihen, että on olemassa QM:n aaltofunktio, jonka sanotaan olevan kuin hiukkanen niissä tapauksissa, joissa matemaattiseen objektiin nimeltä aalto vaikutetaan ja se romahtaa (eksitaatio itse on aalto, mutta epä-lokaali tai lokaali operaatio siihen ei ole aina aaltomekaaninen, mikä olisi aina lokaalia). Eksitaatioita siellä ja täällä on tietysti parempi nimitys asialle.

        QFT ei ole vain kentän eksitaatio, vaan siinä on toisistaan täysin erotetttavien eksitaatioiden olemista ja ei-olemista kuvaava matemaattinen objekti nimeltä operaattori. ja jokaisen sellaisen vaikuttamista yhdessä avaruuden pisteessä (joko vakuumiin tai siihen mitä siellä on) pidetään myös sinä että, jokaisessa avaruuden pisteessä on joukko operaattoreita eli matemaattinen objekti eli kenttä, joka onkin Q-kenttä. Tällöin asia kuten vetovoima tai hylkimisvoima voi saada hiukkasluonteen virtuaalihiukkasina, mutta sekin tarkoittaa eksitaatioita. Vuorovaikutus pitää jakaa näihin eksitaatioihin, joita on jokin laskettava kokonaisluku-määrä aivan kuten hiekan hiukkasia pullossa, ja tässä on paljon kreikkalaisen reduktionismin (atomismin) piirteitä eikä vain infinitesimaalisen reduktionismin, kuten 1700-1800-luvulla oltiin totuttu.

        Infinitesimaalisessa reduktionismissa ei ole mitään kiinteää, jos tarkoitetaan ettei olisi mitään jakamatonta. Jos on olemassa materiaalifyysikkoja, he osaavat infinitesimaalisin menetelmin kertoa vielä tarkemman määritelmän sille, mikä makrokappaleista tulee olemaan kiinteä ja mikä ei. QFT:ssä kvantit voivat olla jakamattomia ja kiinteitä. Varsinkin kun niiden energian tai muiden suureiden spektri on epäjatkuva ja nimenomaan pistemäinen. Kuten kenttäeksitaatioiden tapauksessa oletetaan aina, jos niillä on sisäinen avaruus, jossa esiintyy aina potentiaali, ja jos se aiheuttaa vain kvanttioskillaatio-ratkaisuja. Kvantissa itsessään esiintyy todennäköisesti jotain infinitesimaalisia matemaattisia objekteja.

        "Kenttäkäsite sinänsä on non-lokaali eli se vaikuttaa joka paikassa "

        Jo klassisissa kentissä kentän pitää vaikuttaa itseensä. Tämä vaikutus on ajateltu vain ajattelemalla vielä lyhyempiä etäisyyksiä ja aikavälejä. Kun nämä välit vaikuttavat vain toisiinsa puhutaan selvästi lokaalisuudesta siinä, miten kenttä muuttuu.

        Toinen tapa ymmärtää kentän vaikuttaminen, on sanoa, että kenttä vaikuttaa vain jos on olemassa toinen objekti. Mikä vaikuttaa kenttään? Mihin kenttä vaikuttaa? Ja millä perusteella kenttä ja objektit kokisivat vaikutuksen mistä ja mihin tahansa?

        1

        Kvanttien on tarkoitus vaikuttaa joka paikassa siten, että yhden elektronin luota lähtee virtuaalinen fotoni kaikkiin avaruuden muihin pisteisiin jollain todennäköisyydellä. Siinä on siis hiukkasia, joista jokin on periaatteessa aina joka paikassa, vaikka ne olivat peräisin hyvin pienestä lähteestä joka oli vain hyvin pienessä paikassa. Mutta näin ei käytännössä tapahdu usein ja teoriaa on välttämätöntä käyttää vain pienillä etäisyyksillä, jotka saavutetaan vain hiukkaskiihdyttimessä.

        "Viimeisen naulan ideologisen reduktionismin eli modernin materialismin arkkuun naulasi delayed-choise-experiment koe joka osoittaa kiistatta että lokaali realismi ei voi pitää paikkaansa."

        Lokalismissa ei ole mitään realistista ei-lokalismiin verrattuna. Eikä se ole yhtään reduktionisempaa (ja esim. delayed-choisen viittaama todellisuus on niin reduktionistista kuin millaiseksi asiantuntijat sen joskus tekisivät sitten, kun heistä asiantuntijoita tulee). Eikä mikään näistä filosofian termeistä liity mihinkään toisista termeistä.

        "Mikään ei tarkkaan ottaen liiku mihinkään eikä mitään pienten biljardipallojen kaltaisia aineen perusyksiköitä eli hiukkasia ole olemassakaan. Oikeastaan fysiikassa voidaan ilmaista asioita vain matemaattisesti - ymmärrettävissä oleva kuvaus ei ole mahdollinen arkikielen ja arkiymmärryksen puitteissa. "Shut up and calculate" on hyvä neuvoa Enqvistille, Valtaojalle ja lukemattomille muille heidän kaltaisilleen. Nykyfysiikka ei kerta kaikkiaan ole kuvattavissa kadunmiehen common-sense terminologialla ilman filosofista tahatonta huumoria tai muuta sekoilua."

        'Matematiikkaa tulisi käyttää vain lopputulosten laskemiseen', koskee mahdollisesti kaikkea ylläolevaa. Se koskee yhtäpaljon sinuakin, eli et voisi alkaa väittelemään, että maailma koostuu eksitaatioista. Tai että eksitaatioilla on meille jokin sanoma ja todistusvoima tiettyjä noita täyisin filosofisia sanoja varten kuten materialismi ja realismi. Eksitaatioita kun ei voida ottaa matematiikan ulkopuolelle, ja pitää niitä todellisuuden rakennelmana perimmäisessä mielessä, jolle se matematiikka olisi tehty sattumalta oikeaksi kuvaukseksi parin pallovirheen jälkeen.

        "Ainoa varma pohja tieteelle ja filosofialle on lähteä rakentamaan maailmankuvaa sellaisesta mikä on ehdottoman varmaa ja kiistatonta - nimittäin tietoisuus ja sen sisällöt."

        Aineelle tehdyt matemaattiset kuvaukset ovat joko ainetta (sitä mitä luontokin tekee) tai ihmisen tietoisuuden sisältöä. Siitä, mistä tässä yritetään kai sanoa, pitäisi vain saada yksi hyvä esimerkki, joka ravisuttaa koko maailmaa eikä vain tietoisuuden valmiita faneja.

        2


    • Järki-ja-usko

      Enqvistin puheita voidaan kritisoida hyvin perustein, mutta on noloa, jos oikeanlaisen kritiikin esittämisen jälkeen esitetään jotakin aivan vääränlaista aiheen ohi menevää kritiikkiä. Enqvist tekee luennoillaan uskontoa - paikan päällä olleena tiedän sen - mutta pitkäaikaisilla mittauksilla aikaansaadussa falsifioitavissa olevassa kuvassa kosmoksesta ei ole mitään uskonnollista. Se on tieteellistä jos mikä. Vain jos siitä kosmoksen mittasuhteita ja liikeratoja käsittelevästä kuvasta yritetään johtaa joitakin teologian tai materialismin lauseita, siirrytään jälleen uskonnon piiriin. Ja niinhän Enqvist tekee. Enqvist käytännössä arvelee, että luennolle saapuneet ovat tulleet sinne alkuräjähdyksen poksauttamina, mutta sellainen erittäin huvittava ja kaikkia tilastollisia todennäköisyyksiä ja sitä myötä järkeä vastaan käyvä uskonnollinen filosofointi on erillään hänen todellisista tieteellisistä kosmologisista tutkimuksistaan.

    • episyklomaniaa

      Tieteen popularisointi on kyllä aika avutonta Suomessa.


      Tässä videossa Valtaoja selittää kaiken leegojen avulla:

      https://www.youtube.com/watch?v=hMf-HynFi1s

      Esitelmä oli ilmeisesti tarkoitettu (ainakin Valtaojan kielikuvista päätellen) lähinnä 5 vuotiaille esikoululaisille.

      Enqvist taas "kunnostautui" joku aika sitten kansainvälisessä paneelissa ja suurin piirtein hänen ainoa kommenttinsa oli että "Kaikkihan tiedämme että kukaan ei ymmärrä kvanttifysiikkaa".

      https://www.youtube.com/watch?v=vHLwEmIrSuk

      Minun nuoruudessani 30-40 v sitten tämäntyyppiset sekoilut eivät menneet läpi mutta nykyään porukka taitaa olla niin tehokkaasti tyhmistettyä että mikä vaan uppoaa kuin häkä kunhan ilmaisijana on joku ns. tieteen guru.

      En ole lukenut ainuttakaan Valtaojan ja Enqvistin kirjaa muutamia lainauksia lukuunottamatta mutta näillä näkymin en tule koskemaan ko. herrojen tuotantoon pitkällä kepilläkään koskaan....

      Joku tästäkin kommentistani varmaan taas vetää herneen nenäänsä....

      • Materialismi-on-uskonto

        Enqvistin paneelikeskustelun "Is modern physics crossing the boundaries of science?" nimi kuvaa hyvin sitä, miten materialismi sekoitetaan yleisesti tieteeseen.

        Moderni fysiikkahan on tiedettä itseään. Se on kyllä ylittänyt materialismin rajat, mutta ei tieteen rajoja. Se pysyy tiukasti tieteellisessä menetelmässä, ja on pelkkää tiedettä. Materialismi taas on pelkkä dogma siitä, että kaikki on materiaa. Se on uskonto ja tulee pysymään uskontona, joka ei milloinkaan muutu.

        Miten siis voidaan kysyä, onko moderni fysiikka ylittämässä tieteen rajat? Kysyäkseen jotakin sellaista täytyy (1) sekoittaa materialismiuskonto tieteeseen, (2) pitää materialismiuskontoa pysyvästi saavutettuna etuna, ikuisesti totena tieteen tuloksena, sen kruununjalokivenä (3) pitää uskontona kaikkea sellaista inhimillistä toimintaa, jonka tulokset ovat ristiriidassa materialismiuskonnon kanssa, vaikka sitä toimintaa harjoitettaisiin tiukimmista tiukimmalla tieteellisellä menetelmällä. Vain siten voidaan mitenkään esittää - edes kysymyksen muodossa - otsikon kaltainen todella rankka aivopieru, jonka mukaan moderni fysiikka on astumassa tieteen ulkopuolelle.


      • Materialismi-on-uskonto kirjoitti:

        Enqvistin paneelikeskustelun "Is modern physics crossing the boundaries of science?" nimi kuvaa hyvin sitä, miten materialismi sekoitetaan yleisesti tieteeseen.

        Moderni fysiikkahan on tiedettä itseään. Se on kyllä ylittänyt materialismin rajat, mutta ei tieteen rajoja. Se pysyy tiukasti tieteellisessä menetelmässä, ja on pelkkää tiedettä. Materialismi taas on pelkkä dogma siitä, että kaikki on materiaa. Se on uskonto ja tulee pysymään uskontona, joka ei milloinkaan muutu.

        Miten siis voidaan kysyä, onko moderni fysiikka ylittämässä tieteen rajat? Kysyäkseen jotakin sellaista täytyy (1) sekoittaa materialismiuskonto tieteeseen, (2) pitää materialismiuskontoa pysyvästi saavutettuna etuna, ikuisesti totena tieteen tuloksena, sen kruununjalokivenä (3) pitää uskontona kaikkea sellaista inhimillistä toimintaa, jonka tulokset ovat ristiriidassa materialismiuskonnon kanssa, vaikka sitä toimintaa harjoitettaisiin tiukimmista tiukimmalla tieteellisellä menetelmällä. Vain siten voidaan mitenkään esittää - edes kysymyksen muodossa - otsikon kaltainen todella rankka aivopieru, jonka mukaan moderni fysiikka on astumassa tieteen ulkopuolelle.

        Tiede ei kysy, mitä joku on, vaan miten se toimii. Sanaa "materia" käytetään rinnan sanan "energia" kanssa. Tunnettu on myös niiden määrällinen vastaavuus. Ei ole pienintäkään syytä yrittää ottaa kantaa siihen, mitä ne ovat. Muusta ei ole havaintoa.

        Tiede operoi vain sellaisilla asioilla, joista on havaintoja. "Ontologiset" kysymykset eivät kuulu tieteeseen, vaikka filosofisesti suuntautuneet kuinka niitä tyrkyttävät. "Materialismi" on filosofien ja muiden uskonnollisesti suuntautuneiden höpsötyksiä.


      • episyklomaniaa
        al-jabr kirjoitti:

        Tiede ei kysy, mitä joku on, vaan miten se toimii. Sanaa "materia" käytetään rinnan sanan "energia" kanssa. Tunnettu on myös niiden määrällinen vastaavuus. Ei ole pienintäkään syytä yrittää ottaa kantaa siihen, mitä ne ovat. Muusta ei ole havaintoa.

        Tiede operoi vain sellaisilla asioilla, joista on havaintoja. "Ontologiset" kysymykset eivät kuulu tieteeseen, vaikka filosofisesti suuntautuneet kuinka niitä tyrkyttävät. "Materialismi" on filosofien ja muiden uskonnollisesti suuntautuneiden höpsötyksiä.

        ""Materialismi" on filosofien ja muiden uskonnollisesti suuntautuneiden höpsötyksiä."

        "Materialismi" onkin oikeastaan hyvin harhaanjohtava nimitys. Itse käytän nykyään lähinnä termiä "ideologinen reduktionismi" erotuksena metodologisesta reduktionismista joka kuuluu luonnontieteiden metodiin.

        Ideologinen reduktionismi on filosofinen ja ontologinen asenne kaikkeen kokemukseen (mukaanlukien aistihavainnot kaiken kokemuksen yhtenä aspektina) ja se vähättelee tietoisuuden merkitystä ja olettaa sen palautuvan ensin aivoihin ja sitä kautta alkeishiukkasiin.


      • episyklomaniaa

        Idealistinen reduktionismi on myös yksittäisen ihmisen ja ihmiskunnan tulevaisuuden kannalta varsin itsetuhoinen ideologia. Sillä ei ole mitään positiivista, myönteistä eikä elämää kannattavaa sanomaa. Johdonmukainen materialisti voi kyllä kuvitella olevansa hyvinkin "tieteellinen" vaikka todellisuudessa materialismi on lapsellista vastareaktiota uskonnon aikaisempaan valta-asemaan.

        Materialismi aiheuttaa itsetuhoisuutta ja mielenterveysongelmia. Looginen johtopäätös materialistisesta maailmankuvasta on päättää elämänsä mahdollisimman nopeasti jotta välttyisi kärsimyksiltä. Henkinen kärsimys on kuitenkin osoitus siitä että kaikki ei ole omassa maailmankuvassa ja arvomaailmassa ihan kunnossa.

        Oikea suunta ei ole ajatella kaikkea loogisesti vastakkaisella tavalla kuin mitä uskonnollisissa opeissa sanotaan vaan kulkea jonkinlaista järkevää keskitietä jossa tieteellä ja tekniikalla on välinearvo ihmisen hyvinvoinnin kannalta. Fysiikkatieteeseen ja alkeishiukkasten liikkeisiin ei perustaa etiikkaa ja oman elämän merkitystä. Tieteestä ei ole juuri mitään apua tehtäessä tärkeitä omaa elämää koskevia valintoja - tiede ei voi tavoittaa yksilöllisen ihmiselämän ainutlaatuisuutta millään tasolla. Tiede voi olla eksaktia mutta samalla hyvin triviaalia ja merkityksetöntä nimenomaan yksilön kannalta.

        Myöntämällä tietoisuuden ensisijaisuus ei jouduta takaisin pimeään keskiaikaan eikä taikauskoon vaan otetaan aikuismaisesti vastuu omasta ja muiden elämästä.

        Ihmiskunta on oppinut tieteen avulla paljon aistimaailmasta mutta oma tietoisuus, minuus ja varsinkin piilotajunta jäänyt monelle täysin vieraaksi - useimmat ihmiset ovat oman piilotajuntansa sätkynukkeja.

        Onnellisuuden tilan voi saavuttaa elämällä nykyhetkessä takertumatta pelkoihin, toiveisiin ja uskomuksiin. Egominuus on aina jonkinlaiseen pelkoon perustuvaa ja nimenomaan se pelko työntää suuren osan tietoisuudesta piiloon.


        Oivallukset ovat aina henkilökohtaisia joten tuskin kukaan ajatusteitse pystyy muuttamaan omaa eikä varsinkaan kenenkään toisen maailmankuvaa. Siihen tarvitaan yleensä joku voimakas emotionaalinen shokki.

        ....


        Nyt jään itse viettämään joulua ja uuttavuotta! Voikaa hyvin ja leikkikää kiltisti.... :-)


    • foliop

      Mistähän voisi ostaa jonkin Himangan kirjan? Esimerkiksi kirjan "Se ei sittenkään pyöri". Luettuani kirjan voisin osalistua keskusteluun.

      • Kyllä-se-pyörii

        Enqvist tekee ideologiaa, ja Himanka töppäsi asiassa jossa Enqvist oli oikeassa.


      • Anonyymi

        Tairat ite olla se herra vilosoohvi nii kai ny ite omat opukses tunnetta ?


    • F.Ilosofi

      Pauli Pylkön kirjoituksessa Enqvistin dilemma on kuvattu hyvin. Enqvist sekä suoranaisesti halveksii filosofiaa ja teologiaa että ryhtyy pätemään filosofian ja teologian alueilla. On kuin Enqvist ei lainkaan tunnistaisi harjoittamaansa toimintaa filosofoinniksi.

      http://www.uunikustannus.fi/fysiikkaviikari.pdf

      • Anonyymi

        Pylkkö puhuu painavaa asiaa Enqvistin "filosofoinnista". Olen itse tullut samankaltaisiin johtopäätöksiin, vaikka arvostan Enqvistin tietämystä fysiikasta.

        Enqvistin filosofiset ilmaisut herättävät samaa myötähäpeää kuin Stephen Hawkingin vastaavat pohdinnat. Tässäkin tapauksessa tiedemiehen ego paisui mittakaavassa, joka ei voinut kuin synnyttää kysymyksen: Miksi?


    • Onpa hauska lukea tätä kuusi vuotta vanhaa keskusteluketjua, kun se heijastelee täysin ajatuksiani Himangasta luettuani hänen uusinta kirjaansa Filosofia ja tämä elämä.

      Himangan kritiikki luonnontieteitä kohtaan on yhtä hataraa kuin aiemminkin, mutta puheet filosofiasta ovat vielä omituisempia. Vaikkapa se Kantin siteeraus, jossa todetaan, että vaikka oikea ja vasen käsi ovat toistensa vastinkappaleita niin oikean käden hansikas ei sovi vasempaan käteen. Tästä päätellään, että "ajatusta tilasta esineiden ominaisuutena ei voi hyväksyä"! Onpa kerrassaan omituinen filosofin tulkinta aivan erilaisista tiloista (oikea käsi on erilainen kuin vasen käsi). Samanlaisia loogisia ja käsitteellisiä sekoiluja kirja on täynnä.

    • Anonyymi

      Filosofin viisauden tunnistaa siitä kun hän itse julkisesti todistaa tyhmyytensä.

      • Anonyymi

        Lausuma on itse asiassa totta. Mitään yhteistä, oletetusti rationaalista tutkimusmenetelmää ei ole; mitään yleisesti hyväksyttyä todellisuus- ja totuuskäsitystä ei ole; mitään ristiriidatonta teoriakokonaisuutta ei ole; mitään selvyyttä siitä, mitä kieltä, matematiikkaa ja logiikkaa olisi käytettävä, ei ole.

        Voi tulla järkytyksenä, jos ei ole filosofiaan perehtynyt, mutta yllä oleva on tieteellisessä tutkimuksessa totta. Silti ei tarvitse romahtaa tieteen vastaisuuteen, vaan tunnustaa se tosiasia, että näin on asiantila.


      • Anonyymi
        Anonyymi kirjoitti:

        Lausuma on itse asiassa totta. Mitään yhteistä, oletetusti rationaalista tutkimusmenetelmää ei ole; mitään yleisesti hyväksyttyä todellisuus- ja totuuskäsitystä ei ole; mitään ristiriidatonta teoriakokonaisuutta ei ole; mitään selvyyttä siitä, mitä kieltä, matematiikkaa ja logiikkaa olisi käytettävä, ei ole.

        Voi tulla järkytyksenä, jos ei ole filosofiaan perehtynyt, mutta yllä oleva on tieteellisessä tutkimuksessa totta. Silti ei tarvitse romahtaa tieteen vastaisuuteen, vaan tunnustaa se tosiasia, että näin on asiantila.

        "Mitään yhteistä, oletetusti rationaalista tutkimusmenetelmää ei ole; mitään yleisesti hyväksyttyä todellisuus- ja totuuskäsitystä ei ole; mitään ristiriidatonta teoriakokonaisuutta ei ole; mitään selvyyttä siitä, mitä kieltä, matematiikkaa ja logiikkaa olisi käytettävä, ei ole."

        Tuo pätee hyvin suhteessa koko tieteen kokonaisuuteen koska tiede ei ole mikään koherenti eli sisäisesti ristiriidaton kokonaisuus vaan varsin sekava joukko arvauksia ja spekulaatioita jotka on tieteen popularisoinneissa löyhästi ja usein varsin epäsuskottavasti linkitetty tosiinsa jonkinlaiseksi korttitaloksi tai "purkkaviritykseksi" eli ns. "tieteelliseksi" maailmankuvaksi.

        Filosofia ei ole mikään erityistiede vaan eräänlaista metatiedettä jossa nimenomaan tutkitaan ja vertaillaan erilaisia teorioita, erilaisia kuvailukieliä ja niiden filosofisia perusoletuksia ja tulkintoja toisiinsa. Jokaisella erityistieteellä ja sen erityistieteen keskenään kiistelevillä suuntauksilla on omat puhtaasti metafysiikkaan eli pohjimmiltaan filosofisiin oletuksiin perustuvat oletuksensa ja tulkintansa todellisuudesta joka ilmenee selkeästi sen tieteenalan käyttämien käsitejärjestelmien, logiikan ja matematiikan kautta.

        Koska tiede on ihmisten suorittamaa inhimillistä toimintaa niin filosofia parhaimmillaan kykenee toimimaan jonkinlaisena terapeuttina ja psykiatrina tai oikeammin sosiaalipsykiatrina koska kaikki alunperin filosofiasta erkaantuneet erityistieteet ovat erikoistuneet ja samalla fragmentoituneet niin pitkälle että niiden tieteiden harjoittajien kokonaiskuva ja maailmankuva on hyvin usein vahvasti vääristynyt ja siinä mielessä voi pahimmassa tapauksessa olla myös vahingollinen ja haitallinen koko ihmislajin kokonaisuuden ja tulevaisuuden kannalta puhummattakaan ympäröivästä luonnosta.

        Himangan kritiikki suhteessa Enqvistiin ja hänen kaltaisiinsa on minusta varsin osuvaa ja pätee suurinpiirtein jokaiseen ns. tiedejulkkikseen jota medioissa hypetetään.
        Niiden ainoa hyödyllinen anti on toimiva ja käyttökelpoinen tekniikka joka tosin mahdollistaa myös monenlaisen väärinkäytön.

        Maailmankuvan kannalta kaikki tieteenalat ovat vahvasti kokonaiskuvaa vääristäviä ja melko alkeellisia todellisuuden hahmotustapoja jotka keskittyvät ainoastaan ns. ulkopuoliseen 3. persoonan fiktiiviseen konsensusnäkökulmaan: siihen mikä ei ole suoraan kokemuksessa vaan pelkästään siitä kokemuksesta käytetyn kielen käsitteiden kautta epäsuorasti pääteltyä ja joidenkin vahvasti ideologisten ja metafyysisten oletusten, ennakkoluulojen ja asenteiden kautta spekuloitua.

        Tiede ja varsinkin matematisoidut luonnontieteet ovat niin vahvasti yliarvostettuja ja kritiikittömästi omaksuttuja ja ehdollistettuja että niistä on muodostunut omanlaisensa fundamentalistinen valtauskonto joka on kaikkia aikaisempia uskontoja paljon vaarallisempi eli suurinpiirtein ajatusviruksena eli meeminä verrattavissa omikron muunnokseen :D


      • Anonyymi
        Anonyymi kirjoitti:

        "Mitään yhteistä, oletetusti rationaalista tutkimusmenetelmää ei ole; mitään yleisesti hyväksyttyä todellisuus- ja totuuskäsitystä ei ole; mitään ristiriidatonta teoriakokonaisuutta ei ole; mitään selvyyttä siitä, mitä kieltä, matematiikkaa ja logiikkaa olisi käytettävä, ei ole."

        Tuo pätee hyvin suhteessa koko tieteen kokonaisuuteen koska tiede ei ole mikään koherenti eli sisäisesti ristiriidaton kokonaisuus vaan varsin sekava joukko arvauksia ja spekulaatioita jotka on tieteen popularisoinneissa löyhästi ja usein varsin epäsuskottavasti linkitetty tosiinsa jonkinlaiseksi korttitaloksi tai "purkkaviritykseksi" eli ns. "tieteelliseksi" maailmankuvaksi.

        Filosofia ei ole mikään erityistiede vaan eräänlaista metatiedettä jossa nimenomaan tutkitaan ja vertaillaan erilaisia teorioita, erilaisia kuvailukieliä ja niiden filosofisia perusoletuksia ja tulkintoja toisiinsa. Jokaisella erityistieteellä ja sen erityistieteen keskenään kiistelevillä suuntauksilla on omat puhtaasti metafysiikkaan eli pohjimmiltaan filosofisiin oletuksiin perustuvat oletuksensa ja tulkintansa todellisuudesta joka ilmenee selkeästi sen tieteenalan käyttämien käsitejärjestelmien, logiikan ja matematiikan kautta.

        Koska tiede on ihmisten suorittamaa inhimillistä toimintaa niin filosofia parhaimmillaan kykenee toimimaan jonkinlaisena terapeuttina ja psykiatrina tai oikeammin sosiaalipsykiatrina koska kaikki alunperin filosofiasta erkaantuneet erityistieteet ovat erikoistuneet ja samalla fragmentoituneet niin pitkälle että niiden tieteiden harjoittajien kokonaiskuva ja maailmankuva on hyvin usein vahvasti vääristynyt ja siinä mielessä voi pahimmassa tapauksessa olla myös vahingollinen ja haitallinen koko ihmislajin kokonaisuuden ja tulevaisuuden kannalta puhummattakaan ympäröivästä luonnosta.

        Himangan kritiikki suhteessa Enqvistiin ja hänen kaltaisiinsa on minusta varsin osuvaa ja pätee suurinpiirtein jokaiseen ns. tiedejulkkikseen jota medioissa hypetetään.
        Niiden ainoa hyödyllinen anti on toimiva ja käyttökelpoinen tekniikka joka tosin mahdollistaa myös monenlaisen väärinkäytön.

        Maailmankuvan kannalta kaikki tieteenalat ovat vahvasti kokonaiskuvaa vääristäviä ja melko alkeellisia todellisuuden hahmotustapoja jotka keskittyvät ainoastaan ns. ulkopuoliseen 3. persoonan fiktiiviseen konsensusnäkökulmaan: siihen mikä ei ole suoraan kokemuksessa vaan pelkästään siitä kokemuksesta käytetyn kielen käsitteiden kautta epäsuorasti pääteltyä ja joidenkin vahvasti ideologisten ja metafyysisten oletusten, ennakkoluulojen ja asenteiden kautta spekuloitua.

        Tiede ja varsinkin matematisoidut luonnontieteet ovat niin vahvasti yliarvostettuja ja kritiikittömästi omaksuttuja ja ehdollistettuja että niistä on muodostunut omanlaisensa fundamentalistinen valtauskonto joka on kaikkia aikaisempia uskontoja paljon vaarallisempi eli suurinpiirtein ajatusviruksena eli meeminä verrattavissa omikron muunnokseen :D

        "Niiden ainoa hyödyllinen anti on toimiva ja käyttökelpoinen tekniikka joka tosin mahdollistaa myös monenlaisen väärinkäytön."

        Mitähän tekniikkaa esim. kvanttimekaniikka on mahdollistanut?
        CPU:t ja pienimmätkin puolijohdetransistorit toimivat puhtaasti Maxwellin yhtälöiden, eli valoeetteriparadigman pohjalta. Niissä ei kvantti-ilmiöt vaikuta ollenkaan. Toimintahäiriötkin johtuvat yleensä siitä että lämpötila nousee liian korkealle, mutta tällöinkään kvantti-ilmiöillä ei ole CPU:n toimintahäiriöiden ilmaantuvuuteen minkäänlaista vaikutusta.
        Nykytekniikka olisi kehittynyt yhtä nopeasti ja yhtä toimivaksi myös ilman Einsteiniä, Plancia, Heisenbergiä ym. modernia fysiikkaa.


      • Anonyymi
        Anonyymi kirjoitti:

        "Mitään yhteistä, oletetusti rationaalista tutkimusmenetelmää ei ole; mitään yleisesti hyväksyttyä todellisuus- ja totuuskäsitystä ei ole; mitään ristiriidatonta teoriakokonaisuutta ei ole; mitään selvyyttä siitä, mitä kieltä, matematiikkaa ja logiikkaa olisi käytettävä, ei ole."

        Tuo pätee hyvin suhteessa koko tieteen kokonaisuuteen koska tiede ei ole mikään koherenti eli sisäisesti ristiriidaton kokonaisuus vaan varsin sekava joukko arvauksia ja spekulaatioita jotka on tieteen popularisoinneissa löyhästi ja usein varsin epäsuskottavasti linkitetty tosiinsa jonkinlaiseksi korttitaloksi tai "purkkaviritykseksi" eli ns. "tieteelliseksi" maailmankuvaksi.

        Filosofia ei ole mikään erityistiede vaan eräänlaista metatiedettä jossa nimenomaan tutkitaan ja vertaillaan erilaisia teorioita, erilaisia kuvailukieliä ja niiden filosofisia perusoletuksia ja tulkintoja toisiinsa. Jokaisella erityistieteellä ja sen erityistieteen keskenään kiistelevillä suuntauksilla on omat puhtaasti metafysiikkaan eli pohjimmiltaan filosofisiin oletuksiin perustuvat oletuksensa ja tulkintansa todellisuudesta joka ilmenee selkeästi sen tieteenalan käyttämien käsitejärjestelmien, logiikan ja matematiikan kautta.

        Koska tiede on ihmisten suorittamaa inhimillistä toimintaa niin filosofia parhaimmillaan kykenee toimimaan jonkinlaisena terapeuttina ja psykiatrina tai oikeammin sosiaalipsykiatrina koska kaikki alunperin filosofiasta erkaantuneet erityistieteet ovat erikoistuneet ja samalla fragmentoituneet niin pitkälle että niiden tieteiden harjoittajien kokonaiskuva ja maailmankuva on hyvin usein vahvasti vääristynyt ja siinä mielessä voi pahimmassa tapauksessa olla myös vahingollinen ja haitallinen koko ihmislajin kokonaisuuden ja tulevaisuuden kannalta puhummattakaan ympäröivästä luonnosta.

        Himangan kritiikki suhteessa Enqvistiin ja hänen kaltaisiinsa on minusta varsin osuvaa ja pätee suurinpiirtein jokaiseen ns. tiedejulkkikseen jota medioissa hypetetään.
        Niiden ainoa hyödyllinen anti on toimiva ja käyttökelpoinen tekniikka joka tosin mahdollistaa myös monenlaisen väärinkäytön.

        Maailmankuvan kannalta kaikki tieteenalat ovat vahvasti kokonaiskuvaa vääristäviä ja melko alkeellisia todellisuuden hahmotustapoja jotka keskittyvät ainoastaan ns. ulkopuoliseen 3. persoonan fiktiiviseen konsensusnäkökulmaan: siihen mikä ei ole suoraan kokemuksessa vaan pelkästään siitä kokemuksesta käytetyn kielen käsitteiden kautta epäsuorasti pääteltyä ja joidenkin vahvasti ideologisten ja metafyysisten oletusten, ennakkoluulojen ja asenteiden kautta spekuloitua.

        Tiede ja varsinkin matematisoidut luonnontieteet ovat niin vahvasti yliarvostettuja ja kritiikittömästi omaksuttuja ja ehdollistettuja että niistä on muodostunut omanlaisensa fundamentalistinen valtauskonto joka on kaikkia aikaisempia uskontoja paljon vaarallisempi eli suurinpiirtein ajatusviruksena eli meeminä verrattavissa omikron muunnokseen :D

        Kommentoijan kanssa olen samaa mieltä, paitsi johtopäätöksestä, että tieteestä olisi tullut valtauskonto. Vaikka voi olla fundamentalistisia tiedemiehiä, niin itse tiede on instituutiona on varsin joustava ja uudistuva menetelmissään. Vaaranpaikka on tieteellisen tiedon soveltamisessa, jossa jatkuvasti aiheutetaan tarkoittamattomia sivuvaikutuksia.

        Tarvittaisiin enemmän filosofeja myös tiedon soveltamisen alueella.


      • Anonyymi
        Anonyymi kirjoitti:

        "Niiden ainoa hyödyllinen anti on toimiva ja käyttökelpoinen tekniikka joka tosin mahdollistaa myös monenlaisen väärinkäytön."

        Mitähän tekniikkaa esim. kvanttimekaniikka on mahdollistanut?
        CPU:t ja pienimmätkin puolijohdetransistorit toimivat puhtaasti Maxwellin yhtälöiden, eli valoeetteriparadigman pohjalta. Niissä ei kvantti-ilmiöt vaikuta ollenkaan. Toimintahäiriötkin johtuvat yleensä siitä että lämpötila nousee liian korkealle, mutta tällöinkään kvantti-ilmiöillä ei ole CPU:n toimintahäiriöiden ilmaantuvuuteen minkäänlaista vaikutusta.
        Nykytekniikka olisi kehittynyt yhtä nopeasti ja yhtä toimivaksi myös ilman Einsteiniä, Plancia, Heisenbergiä ym. modernia fysiikkaa.

        https://en.wikipedia.org/wiki/Applications_of_quantum_mechanics

        Sähköopissa esim. elektronin energia saa mitä tahansa arvoja (kahden arvon väliltä), ja vain kvanttimekaniikassa voi olla kiellettyjä ja sallituja arvoja. Aineiden ominaisuudet perustuvat myös siihen, että elektronit jakautuvat tilastollisesti tällaista kiellettyä aluetta pienempiin ja suurempiin energian arvoihin, mikä perustuu kvanttistatistiikkaan. Myös aineen, missä elektronit ovat ja sen isompi rakenne ja komponenteissa käytetty kristallirakenteen omaava aine on peräisin kvanttimekaniikasta.
        https://en.wikipedia.org/wiki/Electronic_band_structure

        Yhtään käytännön transistoria ei syntynyt ennen kuin otettiin tietoisesti huomioon esim. elektronitilojen muuttuminen johteen pinnalla, missä rakenteen sisällä oleva EM-potentiaalin ratkaisu kohtaa atomisen vakuumin ratkaisun.
        https://en.wikipedia.org/wiki/Semiconductor_device_fabrication
        https://en.wikipedia.org/wiki/Surface_states

        Metallijohtimessa lämpötilan nousu kasvattaa sähkönvastusta (voitko selittää sen maxwellin yhtälöillä? *). Puolijohteista jotkut korkeammat lämpötilat laskevat vastusta (miten samat yhtälöt ottavat puolijohteen tässä huomioon?)

        * https://en.wikipedia.org/wiki/Joule_heating
        https://en.wikipedia.org/wiki/Free_electron_model#Mean_free_path
        https://en.wikipedia.org/wiki/Bloch–Grüneisen_temperature
        Kaikki metallitkin ovat kvanttimekaanisia.


      • Anonyymi
        Anonyymi kirjoitti:

        https://en.wikipedia.org/wiki/Applications_of_quantum_mechanics

        Sähköopissa esim. elektronin energia saa mitä tahansa arvoja (kahden arvon väliltä), ja vain kvanttimekaniikassa voi olla kiellettyjä ja sallituja arvoja. Aineiden ominaisuudet perustuvat myös siihen, että elektronit jakautuvat tilastollisesti tällaista kiellettyä aluetta pienempiin ja suurempiin energian arvoihin, mikä perustuu kvanttistatistiikkaan. Myös aineen, missä elektronit ovat ja sen isompi rakenne ja komponenteissa käytetty kristallirakenteen omaava aine on peräisin kvanttimekaniikasta.
        https://en.wikipedia.org/wiki/Electronic_band_structure

        Yhtään käytännön transistoria ei syntynyt ennen kuin otettiin tietoisesti huomioon esim. elektronitilojen muuttuminen johteen pinnalla, missä rakenteen sisällä oleva EM-potentiaalin ratkaisu kohtaa atomisen vakuumin ratkaisun.
        https://en.wikipedia.org/wiki/Semiconductor_device_fabrication
        https://en.wikipedia.org/wiki/Surface_states

        Metallijohtimessa lämpötilan nousu kasvattaa sähkönvastusta (voitko selittää sen maxwellin yhtälöillä? *). Puolijohteista jotkut korkeammat lämpötilat laskevat vastusta (miten samat yhtälöt ottavat puolijohteen tässä huomioon?)

        * https://en.wikipedia.org/wiki/Joule_heating
        https://en.wikipedia.org/wiki/Free_electron_model#Mean_free_path
        https://en.wikipedia.org/wiki/Bloch–Grüneisen_temperature
        Kaikki metallitkin ovat kvanttimekaanisia.

        Bohrin atomimalli oli riittävä transistorin kehittämiseen, varsinaista kvanttimekaniikkaa ei tarvittu mihinkään.


      • Anonyymi
        Anonyymi kirjoitti:

        Bohrin atomimalli oli riittävä transistorin kehittämiseen, varsinaista kvanttimekaniikkaa ei tarvittu mihinkään.

        Koulussa olevat ihmiset voivat puhua toisilleen Bohrin atomimallista samassa lauseessa kuin puolijohteista, koska malli on ensimmäinen laatuaan, jossa esintyy elektronin kaksi energiatasoa. Malli on silti seurausta valosähköilmiöstä ja sen kvantittumisesta (malli itse ei ole valoa, mutta ilmiön takia atomin piti muuttua tällaiseksi). Se on siten kuin atomin kvanttiteoriaa ennen kvanttimekaniikkaa, joka antaa vain vääriä tuloksia. Esim. malli on lähellä oikeita tapauksia atomille, jota kiertää yksi ulko elektroni. Piillä on neljä ulkoista valenssielektronia. Joskus voi ajatella, että jos jokin magia korottaa piin valenssielektroneista vain yhden konduktioelektroniksi, tämä elektroni olisi silloin vähän aikaa Bohrin atomia kiertävä elektroni. Johteen idea (lisäyksenä edelliseen esim. statistiikkaan) ei ole silti vain elektroni, joka kiertää atomia, vaan myös alimiehitetty valenssivyö pitää ottaa huomioon ja näille elektroneille tarvitaan kiihyvä liike materiaalin läpi, missä yksinkertaisin ratkaisu on täysin kvanttimekaaninen.


    • Anonyymi

      "Kaksosparadoksia ei hänen mielestään voi selittää kiihtyvän liikkeen avulla, koska kiihtyvyyttä ei voi mitata, sillä "Einsteinin hississähän me nimenomaan emme tiedä olemmeko kiihtyvässä liikkeessä vai vetovoimakentässä.""

      Oletus 1: Kasoisparadoksitilanteen "voi luoda" gravitaatiolla, missä avaruus kaareutuu pienempään potentiaaliin jossain maan ulkopuolella vetäen toisen kaksosen ulospäin. Ja sitten avaruus kaareutuu päinvastaiseen suuntaan ja kaksonen palaa takaisin maahan.

      Oletus 2: Einsteinin hissi on totta. Kiihtyvyyttä ja avaruuden kaareutuvuutta ei voida erottaa toisistaan.

      Toteamus (1 ja 2 => ): Kaksoisparadoksitilanteen "voi luoda" kiihtyvän liikkeen avulla.

      • Anonyymi

        Niin, Himanka ei näytä ymmärtävän, että kiihtyvyyden muutoksen voi havaita.

        Vaikka oletus 2 on totta: kiihtyvyyttä ja avaruuden kaarevuutta (gravitaatiovoiman vaikutusta) ei voida erottaa toisistaan, silti voidaan mitata, miten kappaleen kiihtyvyys muuttuu inertiaa mittamalla. Näin voimme varmuudella selvittää, kumpi kaksosista lähtee liikkeelle lähtöpaikastaan.


      • Anonyymi
        Anonyymi kirjoitti:

        Niin, Himanka ei näytä ymmärtävän, että kiihtyvyyden muutoksen voi havaita.

        Vaikka oletus 2 on totta: kiihtyvyyttä ja avaruuden kaarevuutta (gravitaatiovoiman vaikutusta) ei voida erottaa toisistaan, silti voidaan mitata, miten kappaleen kiihtyvyys muuttuu inertiaa mittamalla. Näin voimme varmuudella selvittää, kumpi kaksosista lähtee liikkeelle lähtöpaikastaan.

        Kyseessä ei ole kiihtyvyyden muutos kappaleelle."Paradoksi syntyy" jos kiihtyvyydessä on vain yksi muutos edestä taaksepäiseksi kiihtyvyydeksi. Mahdollisesti välissä on yksi nollakiihtyvyyden aika, mutta se ja alun ja lopun nollakiihtyvyydet ovat silti turhia. Kahden eri kappaleen (sekä koordinaatistojen, jotka kiihtyvät) kiihtyvyyttä voidaan verrata lähinnä teoriassa, ja huomata että niiden välillä on "ero". Seuraavassa on eron näkeminen käytännössä.

        Jos tiedetään kumpi kaksonen on kumpi, on mahdollista tietää kumpi lähti liikkeelle, jos kummin etäisyys kampiin muuttuu. Inertiaalikoordinaatistoissa on kaikissa tällöin joku etäisyyden muutos. Liikkeelle lähteminen nopeudesta, joka olisi ollut tähän asti sama kummilla ja kammilla, on tosiasiassa nopeuden muutos eli kiihtyvyys. Onko silti joku syy ottaa selvää hänen liikkeestään ennen kuin se tapahtuu havaittavasti, eli odottamalla vain lyhyt aikaväli (missä joku paljon kaksosta kevyempi kappale ehtii reagoida tms.)?


      • Anonyymi
        Anonyymi kirjoitti:

        Kyseessä ei ole kiihtyvyyden muutos kappaleelle."Paradoksi syntyy" jos kiihtyvyydessä on vain yksi muutos edestä taaksepäiseksi kiihtyvyydeksi. Mahdollisesti välissä on yksi nollakiihtyvyyden aika, mutta se ja alun ja lopun nollakiihtyvyydet ovat silti turhia. Kahden eri kappaleen (sekä koordinaatistojen, jotka kiihtyvät) kiihtyvyyttä voidaan verrata lähinnä teoriassa, ja huomata että niiden välillä on "ero". Seuraavassa on eron näkeminen käytännössä.

        Jos tiedetään kumpi kaksonen on kumpi, on mahdollista tietää kumpi lähti liikkeelle, jos kummin etäisyys kampiin muuttuu. Inertiaalikoordinaatistoissa on kaikissa tällöin joku etäisyyden muutos. Liikkeelle lähteminen nopeudesta, joka olisi ollut tähän asti sama kummilla ja kammilla, on tosiasiassa nopeuden muutos eli kiihtyvyys. Onko silti joku syy ottaa selvää hänen liikkeestään ennen kuin se tapahtuu havaittavasti, eli odottamalla vain lyhyt aikaväli (missä joku paljon kaksosta kevyempi kappale ehtii reagoida tms.)?

        Kumpi kaksosista lähti matkaan? kyselee Himanka kaksosparadoksia pohtiessaan.

        Se on mahdollista selvittää esim. kun kaksosista toinen laitetaan rakettiin, jota kiihdytetään niin suureen kiihtyvyyteen, että G-voimat murskaavat astronautin hengiltä. Tällöin kuollut kaksonen on se, joka lähti matkaan. Elossa oleva jäi odottamaan toverinsa paluuta. MOT


      • Anonyymi
        Anonyymi kirjoitti:

        Kumpi kaksosista lähti matkaan? kyselee Himanka kaksosparadoksia pohtiessaan.

        Se on mahdollista selvittää esim. kun kaksosista toinen laitetaan rakettiin, jota kiihdytetään niin suureen kiihtyvyyteen, että G-voimat murskaavat astronautin hengiltä. Tällöin kuollut kaksonen on se, joka lähti matkaan. Elossa oleva jäi odottamaan toverinsa paluuta. MOT

        Kumpi lähti matkaan on samantekevää pohtia tässä paradoksissa tai se on aina harvinaisen selvää. Kerron kaikista tavoista ottaa liikkumisesta selvää, mutta ensimmäisen käsitteen seuraus on juuri se ettei kysymys voi vaikuttaa tämän paradoksin käsittelyyn. Ilmeisesti pohdinta nousee siitä, että molemmat kaksoset liikuvat toisiinsa nähden, eli molempien voi sanoa liikkuvan. Mitä ajatuksia tästä pitäisi seurata (ja miksi sitten tulisi mainita kiihtyvyys ja gravitaatio ja niiden ekvivalenssi tapaus)?

        Jos mennään takaisin alkuun, missä paradoksia ei muodosteta kiihdytyksen avulla, on olemassa systeemi, joka pätee myös kaikkiin muihin tapauksiin. On olemassa lähtöpiste A, kaksi kaksosta, ja on olemassa lisäksi piste B, jossa toinen kaksonen käy ja josta hän palaa takaisin. Liikkumaton kaksonen on liikkumaton pisteisiin A ja B nähden (ne ovat mieluiten hänen koordinaatistossaan valittuja pisteitä alussa). Liikkuva kaksoneen on liikkeessä kohti pistettä B. Hänen ensimmäisessä koordinaatistossaan, piste B on lähempänä A:ta kuin mitä veli aluksi määrittelee (ilman kiihtyvyyttä sanotaan että hänen paluukoordinaatistonsa on uusi kolmas koordinaatisto, joka palaa kohti A:ta taianomaisesti jonkun peilin kautta).

        On siis aina olemassa kaksi täysin erilaista kaksosta, vaikka kiihtyvyyttä ei käytetä. Jos kiihtyvyys otetaan käyttöön, on edelleen piste ensimmäisen kaksosen koordinaatistossa, jossa toinen heistä hänen mukaansa käy kääntymässä. Jos kolmas havaitsija (liikkumaton A:ssa) ei tiedä kaksosten sijainneista ja nopeuksista itse mitään, mutta voi puhua heille hetkellä (t=0, toisen kaksosen nopeus täysi), hän voi kysyä, miten kaukana pisteet A ja B ovat heidän mielestään toisistaan.

        ---

        Kun paradoksin systeemi on korotettu siten, että se luodaan yleisen suhteellisuusteorian avulla, voitaisiin kaksosia verrata toisiinsa siten, että mitataan (teoriassa) avaruuden metriikka siellä, missä he ovat. Se, joka on minkowskin avaruudessa ei ole liikkuva kaksonen. Kaksonen voi mitata tätä itsekin ja saada yhden ja saman tiedon, josta he silloin tunnistavat itsensä.

        Kiihdytyksiä on teoriassa esim. kolmenlaisia. Periaatteessa kiihtymistä on se, että jonkun nopeus jonkun koordinaatistossa muuttuu. Lisäksi on vapaa pudotus, joka saisi äskeisen aikaan, jos vapaa pudotus ei pidä nopeutta vakiona. Lisäksi kiihtyvyydeksi voidaan sanoa kaikkea, mikä estää nimenomaan vapaata pudotusta (jos vapaa pudotus pitäisi nopeuden vakiona tämä kiihdytys olisi se, mikä saisi kiihtymisen ensimmäisessä merkityksessä).

        Vapaa pudotus on sellaista, mikä ei saa aikaan lihavien objektien puristumista kahden voiman väliin, ja kappale voi silti joidenkin mukaan kiihtyä uuteen nopeuteen. Kun ihmiset puristuvat oman painonsa alla maata vasten, heidän vapaapudotuksensa olisi jatkaa matkaa maan keskipiseeseen. Sähkömagneettiset voimat kuitenkin kiihdyttävät heitä päinvastaiseen suuntaan. Samoin avaruusaluksessa vapaapudotus olisi kellua avaruudessa samalla nopeudella kuin äsken. Jos alus käyttää moottoria, voimat kuitenkin estävät ihmisten paikkallaanolemisen ja puristavat häntä lyttyyn. Puristuminen on siis aina merkki siitä, että vapaata putoamista gravitaatiokentässä häiritään oli kyseessä sitten muiden havaitsema kiiihtyminen tai ei.

        Jos katsellaan kiihtyvää objektia omassa koordinaatistossamme, on tapa ottaa selvää, kuka on oikeasti kiihtyvässä liikkeessä, kun ollaan joko litteässä avaruudessa (tai jotenkin samanarvoisessa pisteessä erikoisempaa avaruutta). Siinä muodostetaan objekteja, joiden tiedetään olevan vapaita kaikista ulkoisista voimista (koska ne ovat sähköneutraaleja ja tyhjyyden ympäröimiä) ja katsotaan, miten ne putoavat. Jos ne eivät jää katsojaan nähden paikalleen, on katsoja se joka kiihtyy (mutta sen on oltava tavalla joka on vapaatapudotusta vastustavaa, kuten jos pudotat kirjan lattialle).

        Kaksoisparadoksin sisältänät eri kellonajat eivät perustu siihen, että on olemassa lihavia kappaleita, joita puristetaan, vaan kaikissa tapauksissa luoda se toimii ennenkaikkea kaksosten välinen nopeusero ja aikadilataatio per lyhyt hetki, joka ei ota kantaa kiihtymiseen. Kiihtymiset ja gravitaatiot aiheuttaisivat myös pieniä muutoksia kelloihin, mutta eivät yhtä merkittäviä. Puristumista ei siis voisi pitää merkkinä kaksoisparadoksin tapahtumisesta lähiaikoina ja yllä oleva gravitaatioluotu paradoksi on täysin vapaapudotusta. Kun sanotaan, että kiihtyvyys ja gravitaatio ovat kuin yhtä, tämä toimii tässä paradoksissa aina siten, että koska paradoksi ottaa hyvin vähän kantaa muuhun kuin nopeusaikadilataatioon, sillä ei ole merkitystä, mistä nopeusmuutos tuli ja oliko se tyyppiä joka puristelee kaksosta. Tosiasiassa eksoottisilla avaruuksilla voi olla gravitaatioita, jotka ovat aina kiihtyvyydelle käsittämättömiä.


    • Anonyymi

      Fysiikan alkeita tietämättömille on ehkä hyvä kertoa, mistä tiedämme maapallon pyörivän akselinsa ympäri. Epätosi on siis sen negaatio, että maapallo ei pyöri, vaan kaikki muut taivaankappaleet pyörisivät vinhasti Maan ympärillä ja vain vaikuttaisi siltä, että maapallo pyörisi.

      Selitys on sama kuin, mistä tiedämme pesukoneen rummun pyörivän, kun se linkoaa pyykkiä. Inertian vaikutuksesta pyykki painautuu rummun kehää vasten, kun rumpu pyörii. Tämä fysiikan perusteisiin kuuluva ilmiö saa pyykin kosteuden vähenemään, kun pyörivästä rummusta päästetään pyykistä irtoava vesi pois.

      Maapallon pyöriminen akselinsa ympäri on yhtä tosi väite kuin pesukoneen rummun pyöriminen. Maan pyörimisen aiheuttama inertia voidaan mitata. Siitä tiedetään, että maapallo absoluuttisesti pyörii, eikä sen (pyörimis)liike siinä mielessä ole suhteellista kaikkien muiden taivaankappleiden liikkeeseen.

    • Anonyymi

      "mutta Himanka ei tunnu tajuavan, että avaruusalus EI ole Einsteinin hissi: esimerkiksi kiihdytyksessä astronautti painautuu tuoliaan vasten ja takuuvarmasti tuntee kiihtyvyyden."

      Lukiko kukaan kommentoijista ollenkaan ajatuksella kommentoimaansa tekstiä kokonaan?

      "Vielä kerran asiaa harkittuani huomaan,
      että näkemykseni ei tässä ole johdonmukainen.
      Enqvist on oikeassa. Kiitän huomautuksesta ja
      korjaan kantaani: Einsteinin kannalta maa- ja
      aurinkokeskeinen malli ovat nimenomaan aivan
      yhtä päteviä tai epäpäteviä."

      (3645-Artikkelin teksti-8280-1-10-20101026 s 52 alkup.)

      Tuossa Himanka siis myöntää olleensa väärässä vaikka onkin sitä mieltä että sekä maa- että aurinkokeskeinen malli ovat molemmat vääriä tarkemmin tutkittuna (esim. Aurinko kiertää galaksin keskustaa ja kiertoaika on 230 miljoonaa vuotta)

    • Anonyymi

      Mm. teologiaa "tieteenä" ihaileva rohvessori Juha Himanka on julkisuutta rakastava pelle, joka tietää, että mitä älyttömämpiä sammakoita laukoo sitä enemmän julkisuutta. Ja hädin tuskin lukutaitoisiin toimittajiin saa tänään uppoamaan ihan mitä tahansa - kun okiein rohvessori sanoo.....
      Ylen Radio 1:n kulttuuriykkönen oli tänään kuunneltavaa, joka puppu sai voimaan pahoin. Empiirinen tiede on pelkkää sontaa ja valetta.....

      Ei voi sanoa kuin: Oulun yliopisto!

      Maapallo se pyörii sittenkin!

    • Anonyymi

      Niin no, väittäähän ne planeettamme kuumenevan ennätys tahtia ja kohta ollaan saunan ylimmällä hyllyllä jos ei vihreät pikapikaa saa nostettua polttoaineiden hintoja pilviin.

    • Anonyymi

      Filosofian tohtori Himanka:

      - Ei ymmärrä, että Maa kiertää Aurinkoa gravitaation takia.

      - Ei ymmärrä, että ympyräliikkeessä oleva kappale kokee kiihtyvyyttä eikä siten kulje tasaisella nopeudella ja aiheuta Lenardin olettamaa paradoksia suppeassa suhteellisuusteoriassa.

      - Ei ymmärrä, että Einsteinin päätös valita valon nopeuden vakioisuus suppean suhteellisuusteorian luonnolliseksi peruslähtökohdaksi ei ole mikään hatusta vedetty idea tai ideologinen oletus vaan se perustuu Maxwellin elektrodynamiikkaan, joka on miljoonin kokein varmennettu fysikaalinen teoria ja jossa sähkömagneettisen aallon nopeus on vakio jokaisen lepokoordinaatiston suhteen.

      - Ei ymmärrä, että "kaksosparadoksilla" on selkeä Minkowskin metriikkaan ja aika-avaruudessa kuljettuun matkaan ja sen itseisaikaan perustuva selitys, jossa Maahan jääneen kaksosen kulkeman matka on ajan suhteen pidempi.

      - Ei ymmärrä, että on tehnyt itsestään akateemisen pellen astuessaan ulos omalta huuhaatieteen pätevyysalueeltaan ja lähtiessään väittelyyn luonnontieteilijöiden kanssa.

      • Anonyymi

        Tainnut olla tohtori Himanka kyseisen aiheen tiimoilta aika hiljaa viime vuodet. Moka mikä moka, mutta ei edes filosofin pitäisi noin pahasti ampua jalkaansa.


      • Anonyymi
        Anonyymi kirjoitti:

        Tainnut olla tohtori Himanka kyseisen aiheen tiimoilta aika hiljaa viime vuodet. Moka mikä moka, mutta ei edes filosofin pitäisi noin pahasti ampua jalkaansa.

        Vielä mitä! Vastahan noin puoli vuotta sitten tämä aikamme Sokrates yritti kammeta Einsteinin suhteellisuusteoriaa jalustaltaan: www.youtube.com/watch?v=thZn1O-aXPw

        Vaikuttaa siltä, että Himanka on katkeroitunut - ja pahasti - ja yrittää siksi löytää edes jonkin tavan, jolla tölväistä Einsteiniä sekä modernin fysiikan epäfilosofista maailmanselitystä. Monet muutkin filosofit ovat käärmeissään, kun heidät on ajettu maailmanselittäjinä paitsioon, mutta he sentään ymmärtävät pitää suunsa kiinni.


      • Anonyymi
        Anonyymi kirjoitti:

        Tainnut olla tohtori Himanka kyseisen aiheen tiimoilta aika hiljaa viime vuodet. Moka mikä moka, mutta ei edes filosofin pitäisi noin pahasti ampua jalkaansa.

        Puolustan Himankaa siinä, että jokainen filosofi saa ampua itseään jalkaan tai mihin tahansa muuhunkin ruumiinosaan. Se on jokaikisen filosofin perusoikeus. (En usko, että lukijat tätä ymmärtävät.)

        Mutta fyysikko ei saa ampua itseään jalkaan filosofiassa. Kuten Enqvist tekee, se on perin noloa.


      • Anonyymi
        Anonyymi kirjoitti:

        Puolustan Himankaa siinä, että jokainen filosofi saa ampua itseään jalkaan tai mihin tahansa muuhunkin ruumiinosaan. Se on jokaikisen filosofin perusoikeus. (En usko, että lukijat tätä ymmärtävät.)

        Mutta fyysikko ei saa ampua itseään jalkaan filosofiassa. Kuten Enqvist tekee, se on perin noloa.

        Liittyisikö tämä Niiniluodon toteamukseen, joka oli ajatuksena suunnilleen tällainen:
        Filosofian kannalta teoria saattaa olla täysin validi ilman, että sillä on mitään yhtymäkohtaa reaalitodellisuuteen.
        Taisi olla kirjassa "Totuuden rakastaminen", jossa Niiniluoto ruoti mm. relativismia.


      • Anonyymi
        Anonyymi kirjoitti:

        Vielä mitä! Vastahan noin puoli vuotta sitten tämä aikamme Sokrates yritti kammeta Einsteinin suhteellisuusteoriaa jalustaltaan: www.youtube.com/watch?v=thZn1O-aXPw

        Vaikuttaa siltä, että Himanka on katkeroitunut - ja pahasti - ja yrittää siksi löytää edes jonkin tavan, jolla tölväistä Einsteiniä sekä modernin fysiikan epäfilosofista maailmanselitystä. Monet muutkin filosofit ovat käärmeissään, kun heidät on ajettu maailmanselittäjinä paitsioon, mutta he sentään ymmärtävät pitää suunsa kiinni.

        Katsoin tuon videon, jossa Himanka paasaa suhteellisuusteorian ideologisuudesta ja yrittää työntää käärmettä pyssyyn vähätellessään Einsteiniä - ja samalla koko nykyistä fyysikkokuntaa. Erityisen noloa oli tuo maapallon kiertoliikkeen avulla perusteltu suhteellisuusteorian "kumoaminen", sillä johan nyt sen jo pieni lapsikin ymmärtää, ettei ympyräliike ole suoraviivaista, kuten Einsteinin suhteellisuusteoria edellyttäisi. Himanka olisi voinut vaikka googlettaa aiheesta tai kysyä esim. physics/philosophy stack exchange -foorumeilta. Nyt hän on kuitenkin luottanut omaan erinomaisuuteensa ja lyönyt kirveensä kunnolla kiveen.

        Suurempi periaatteellinen ongelma Himangalla on kuitenkin se, että vaikka hän itse edustaa fenomenologista koulukuntaa, niin hän kuitenkin puolustaa Lorentzin eetteriä, joka on jotakin sellaista jota ei voi kokeellisesti havaita. Siispä, jos jokin substanssi ei ole osa ihmisen havaintomaailmaa, niin silloin sitä substanssia ei myöskään pidä olla olemassa. Vaikka on ehkä hyväksyttävää, ettei filosofi ymmärrä modernia fysiikkaa, niin on suoranainen kuolemansynti jos fenomenologi puolustaa eetterin olemassaoloa!

        Itse asiassa, jos valon nopeus olisi huomattavasti pienempi kuin n. 300 miljoonaa metriä sekunnissa, niin Einsteinin oletus sen invarianssista olisi fenomenologillekin hyväksyttävä, kun havaitsija pääsisi omassa arkielämässään kokemaan liiketilan aiheuttaman dilataation.

        Lisäksi haluan sanoa sanaisen aiemmasta Ptolemaioksen episykliteorian puolustamisesta, johon Himanka on sortunut. Vaikka maakeskeinen aurinkokuntamalli voi selittää planeettojen liikkeet episyklien avulla, niin kahden eri planeetalla sijaitsevan havaitsijan tapaksessa episyklit johtavat fenomenologisessa mielessä ristiriitaan: sekä Maassa että Marsissa sijaitsevien havaitsijoiden mielestä kaikki muut planeetat - kuin oma - pyörivät kuin pallot pesukoneessa. Mutta jos oletetaan todellinen painovoimaan perustuva aurinkokuntamalli, niin sekä Maassa että Marsissa tehdyt havainnot ovat harmonisessa sopusoinnussa, kun kaikki planeetat kiertävät yhteistä massakeskipistettä. Enqvist tuota taisi jo selittää, mutta se kaikui laukalle lähteneellä filosofilla kuuroille korville.

        Vaikka olen aina sanonut, että kenelläkään ei ole oikeutta tukkia filosofin suuta, niin suosittelen kuitenkin kollega Himankaa lopettamaan tämän jo n. 20 vuotta kestäneen tuulimyllyjä vastaan taistelemisen.


    • Anonyymi

      Minua aina huvittaa nämä fyysikot tms., jotka julkisuudessa kertovat massoille, että suhteellisuusteoriat ovat oikeiksi todistettuja, vaikka aikaa ei ole edes olemassa tieteenfilosofisesti.

      Ylipäänsä minua huvittaa, kun joku yrittää väittää, että tiede voi varmuudella sanoa, miten asiat ovat, vaikka moderni matematiikka kertoo meille, ettei 1 1=2 pidä paikkaansa kuin erityistapauksissa.

      Minua huvittaa siis tieteisuskovaisuus, joka ei poikkea millään tavoin muusta uskovaisuudesta.

      Kyllä Himanka aina kaikenmaailman Himaset voittaa mennen tullen, mitä tulee filosofiseen osaamiseen.

      • Anonyymi

        Esimerkiksi suhteellisuusteoriaa on yritetty kaataa jo toista sataa vuotta, mutta vielä kukaan ei ole onnistunut. Sen sijaan esimerkiksi GPS ei toimisi, jos suhteellisuusteorian mukaista gravitaation vaikutusta aikaan ei otettaisi huomioon.
        Filosofialla ei ole mitään annettavaa oikealle tieteelle.


      • Anonyymi
        Anonyymi kirjoitti:

        Esimerkiksi suhteellisuusteoriaa on yritetty kaataa jo toista sataa vuotta, mutta vielä kukaan ei ole onnistunut. Sen sijaan esimerkiksi GPS ei toimisi, jos suhteellisuusteorian mukaista gravitaation vaikutusta aikaan ei otettaisi huomioon.
        Filosofialla ei ole mitään annettavaa oikealle tieteelle.

        Suhteellisuusteoriat kaatuvat toteamalla, että aikaa ei ole olemassa, mikä on itsestään selvyys.


      • Anonyymi
        Anonyymi kirjoitti:

        Suhteellisuusteoriat kaatuvat toteamalla, että aikaa ei ole olemassa, mikä on itsestään selvyys.

        "Suhteellisuusteoriat kaatuvat toteamalla, että aikaa ei ole olemassa"

        Ehkä hieman ennenaikaista julistaa moista.

        Nobelin palkinto Einsteinille 1921 oli raskas isku kyseiselle henkilölle,
        hän oli tullut kuuluisaksi suhteellisuusteorioistaan, mutta niistä ei
        tullut Nobelia, vaan jostain valo-sähköisestä ilmiöstä.

        Nobelkomitean edustajan lausunto oli aika hauska, " Eräs filosofi oli
        Pariisissa langettanut varjon suhteellisusteorioiden ylle, kyse ei ole
        tieteestä vaan filosofisesta näkemyksestä, aikaa ei ole siinä mielessä
        kuin suhteellisuusteorioissa kerrotaan".

        Einstein palkintopuheessaan keskittyi suhteellisuusteorioihinsa aivan
        kuin hän olisi saanut palkinnon niistä.
        Vielä viimeisinä elinvuosinaan Einstein kirjeissään ystävilleen suhtautui
        kyseiseen filosofiin hyvinkin kielteisesti.

        Toisaalta myös Lorentz ja Poicare olivat sitä mieltä että suhteellisuus-
        teoriat ovat lähinnä filosofiaa.

        R.


      • Anonyymi
        Anonyymi kirjoitti:

        Suhteellisuusteoriat kaatuvat toteamalla, että aikaa ei ole olemassa, mikä on itsestään selvyys.

        Tietenkin aika on olemassa, se ei vaan kulje samaan tahtiin joka paikassa.
        "Aikaa ei ole olemassa" on taas näitä filosofisia tyhjäpäisyyksiä, joilla ei ole mitään tekemistä tieteen kanssa. Jos aikaa ei ole olemassa, miten sitä on mahdollista mitata?


      • Anonyymi

        "Minua aina huvittaa nämä fyysikot tms., jotka julkisuudessa kertovat massoille, että suhteellisuusteoriat ovat oikeiksi todistettuja, vaikka aikaa ei ole edes olemassa tieteenfilosofisesti."

        Tieteenfilosofia ei ole instituutio, joka kertoo, onko aika olemassa. Se on filosofien tapa ymmärtää miten tiedettä on tehty, ja on yleensä sata vuotta tiedettä jäljessä, koska vaatii tieteen yleistymistä ja sen kommentoinnin etenemistä suusta toiseen. Jotkut kuitenkin antavat olettaa, että tieteen filosofia voisi antaa paremmat kriteerit ja tukevan alustan ja ohjeet sille, miten tiedettä pitäisi nyt ja tulevaisuudessa tehdä. Tällainen ei minusta silti voi alentua siihen, että ottaisi suoraan kantaa aikaan. Ajasta puhujat voivat olla kuitenkin todellisuudesta puhuvia filosofeja, joiden kannattaisi nimetä itsensä sen menetelmän ja/tai tietyn suuntauksen mukaan, jolla väittävät esim. ajan olevan pois kyseestä.

        Onko olemassa listaa asioista, jotka voivat olla olemassa, vaikka aikaa ei ole olemassa? Oletko aivan varmasti oikeassa, etteivät juuri suhteellisuusteoriat kuulu sille listalle myös ja ensimmäisten joukkoon?

        "Ylipäänsä minua huvittaa, kun joku yrittää väittää, että tiede voi varmuudella sanoa, miten asiat ovat, vaikka moderni matematiikka kertoo meille, ettei 1 1=2 pidä paikkaansa kuin erityistapauksissa."

        Luonnossa ei välttämättä esiinny koskaan yleisiä tapauksia mistään matemaattisesta asiasta. Varsinkaan luonnon tapaukset eivät ole yleisimmällä logiikan tasolla, oli kyseessä sitten esim. joukkojen logiikka tai mitä jos yleistäisit sen, mitä olet filosofiasta oppinut logiisista mahdollisuuksita ja koettaisit löytää sille tai sen alatapauksille kaikille vastaavuuden luonnontieteistä. Niin voitko lopulta väittää päinvastaista, että luonto toimii kaiken yleisenä tapauksena tai sisältää kaikki tapaukset? Miksi filosofit silloin väittelevät mistään asiasta?


      • Anonyymi
        Anonyymi kirjoitti:

        Tietenkin aika on olemassa, se ei vaan kulje samaan tahtiin joka paikassa.
        "Aikaa ei ole olemassa" on taas näitä filosofisia tyhjäpäisyyksiä, joilla ei ole mitään tekemistä tieteen kanssa. Jos aikaa ei ole olemassa, miten sitä on mahdollista mitata?

        "Jos aikaa ei ole olemassa, miten sitä on mahdollista mitata?"

        Miten mittaat aikaa, kelloilla?

        Kellot ovat vain sekuntiannostelijoita.


      • Anonyymi
        Anonyymi kirjoitti:

        "Jos aikaa ei ole olemassa, miten sitä on mahdollista mitata?"

        Miten mittaat aikaa, kelloilla?

        Kellot ovat vain sekuntiannostelijoita.

        Kyllä. Kellot ovat sekuntiannostelijoita, kellolle sekunti on aina saman mittainen sen omassa koordinaatistossa.
        Sekunti on eri mittainen ainoastaan verrattuna ulkopuolelta toisen koordinaatiston kelloon. Tuon yksinkertaisemmin en osaa selittää, jos et ymmärrä niin ehkä kumminkin arvaat mistä vikaa kannattaa etsiä.


      • Anonyymi
        Anonyymi kirjoitti:

        Kyllä. Kellot ovat sekuntiannostelijoita, kellolle sekunti on aina saman mittainen sen omassa koordinaatistossa.
        Sekunti on eri mittainen ainoastaan verrattuna ulkopuolelta toisen koordinaatiston kelloon. Tuon yksinkertaisemmin en osaa selittää, jos et ymmärrä niin ehkä kumminkin arvaat mistä vikaa kannattaa etsiä.

        "Sekunti on eri mittainen...."

        Sinulla on siis hyvä teoria/näkemys miten kellot käyttäytyvät,


      • Anonyymi
        Anonyymi kirjoitti:

        "Sekunti on eri mittainen...."

        Sinulla on siis hyvä teoria/näkemys miten kellot käyttäytyvät,

        Anon62

        Ettet ole vaan sortumassa virhepäätelmään tuossa ajan ja kellojen
        tapauksessa.


    • Anonyymi

      En tiedä osaako Himanka teknistä englantia, mutta tällä videolla selitetään hyvin mistä aika-avaruudessa on kyse, sekä lisäksi miten kaksosten paradoksi tai Newtonin ämpäri -paradoksi eivät oikeasti ole mitään paradokseja. Videolla mainitaan myös sekin asia, jota filosofi Himanka ei ole tuntunut ymmärtäneen: nopeudella on vauhdin lisäksi suunta.

      https://www.youtube.com/watch?v=ZdrZf4lQTSg

    • Anonyymi

      "Onko Juha Himanka uskottava filosofi?"

      Kyllä minä uskon, että sen niminen filosofi on.

    • Anonyymi

      Onneton Himanka on paasannut vuosikaudet Lorenzin eetteristä eikä ole ollut tietoinen, että myös Lorenz itse luopui koko ideasta ja hyväksyi suhteellisuusperiaatteen!

      Seuraavan videon alussa Paul Dirac mainitsee, että lopulta noin vuonna 1909 Lorenz kallistui Einstein kannalle: https://www.youtube.com/watch?v=xJzrU38pGWc

      Olisi kiva tietää, vieläkö Himanka aikoo jatkaa sotaansa tuulimyllyjä vastaan?

      • Anonyymi

        No ei hyvää päivää! Eikö tuonkin asian oli voinut kysymällä tarkastaa joltakin internetin laadukkaalta keskustelupalstalta? Nyt tämä himanka on taistellut tuulimyllyjä vastaan vuosikaudet, kun on hulluuspäissään luullut olevansa koko muuta tiedemaailmaa viisaampi.

        Hyperventiloivaa myötähäpeää aiheuttaa myös se, että himanka on sekoittanut ympyräliikkeen ja suoraviivaisen liikkeen keskenään! Eipä ihmekään, ettei enqvist saanut viestiään perille, kun vastapuolen ymmärrys on tuolla tasolla.


      • Anonyymi
        Anonyymi kirjoitti:

        No ei hyvää päivää! Eikö tuonkin asian oli voinut kysymällä tarkastaa joltakin internetin laadukkaalta keskustelupalstalta? Nyt tämä himanka on taistellut tuulimyllyjä vastaan vuosikaudet, kun on hulluuspäissään luullut olevansa koko muuta tiedemaailmaa viisaampi.

        Hyperventiloivaa myötähäpeää aiheuttaa myös se, että himanka on sekoittanut ympyräliikkeen ja suoraviivaisen liikkeen keskenään! Eipä ihmekään, ettei enqvist saanut viestiään perille, kun vastapuolen ymmärrys on tuolla tasolla.

        Ei filosofien uhmakkuutta tai ajattelun lennokkuutta sovi parjata. Filosofia on filosofiaa ja luonnontiede on sitten asia erikseen.


    • Anonyymi

      Ei. Hän pölisee filosofisia hiukkasia kuin koskaan tamppaamaton parvekematto.

    • Anonyymi

      "Sama jos joku toinen akateemisessa virassa oleva henkilö alkaisi väittämään, että Maa on litteä tai että Kuu on juustoa."

      Himangan edustamaan fenomenologian kannalta on nimenomaan hyväksyttävää väittää, että Maa on litteä jos oma kokemus siltä tuntuu. Kuuta voi myös pitää juustona niin kauan, kun kukaan ole ole käynyt maistamassa.

      Mutta joo, kuten jo moneen kertaan on tullut selväksi, niin filosofialla ei ole nykyfysiikalle mitään annettavaa. Richard Feynmann kiteytti asian hyvin: Fysiikka hyötyy filosofiasta yhtä paljoin kuin linnut ornitologiasta.

      • Anonyymi

        Miksi filosofiaa edes opetetaan yliopistossa, jos se on noin idioottimasta sekä hyödytöntä? Eikö se riitä, että jokainen saa vapaa-ajallaan lukea ja ajatella mitä lystää?


      • Anonyymi
        Anonyymi kirjoitti:

        Miksi filosofiaa edes opetetaan yliopistossa, jos se on noin idioottimasta sekä hyödytöntä? Eikö se riitä, että jokainen saa vapaa-ajallaan lukea ja ajatella mitä lystää?

        Tuo on kyllä hyvä ja aiheellinen kysymys.


      • Kosmologia on ennen kaikkea filosofiaa ja vasta toissijaisesti tähtitiedettä.

        Universumi voi ihan hyvin olla ikuinen ja ääretön ja alkaminen ja laajeneminen tieteen harharetki. Pelkkä ajankohtainen muoti ilmiö, kestänyt vain 60 vuotta ja kasvattanut kaksi sukupolvea uskomaan matemaattisiin hölmöyksiin ja virheellisiin fysikaalisiin päättelyihin.


      • "Fysiikka hyötyy filosofiasta yhtä paljoin kuin linnut ornitologiasta."

        Hmmm, ornitologia on tiedettä. Sattuikohan Feynmanille lipsahdus?


      • Anonyymi
        inti kirjoitti:

        "Fysiikka hyötyy filosofiasta yhtä paljoin kuin linnut ornitologiasta."

        Hmmm, ornitologia on tiedettä. Sattuikohan Feynmanille lipsahdus?

        Ei lintuja hyödytä yhtään mitään se, mitä me ihmiset puhutaan linnuista. Tuon sitaatin taustalla on tarina Feynmannin lapsuudesta, jolloin hänen isänsä sanoi, että voit tietää jonkin lintulajin nimen monella eri kielellä, muttet silti tiedä mitään koko linnusta.


    • Hyvä että hyökätään alkupamausteoriaa vastaan. Se on mahdoton. Antaa väärän kuvan universumista. Uusi teoria edellyttää korjauksia myös suhteellisuusteoriaan.

      • Anonyymi

        Mitä se huru-ukko höpäjää? Avaruuden havaittu laajeneminen viittaa siihen, että kaukana historiassa avaruus on ollu kaikilta etäisyyksiltään erittäin pieni sekä materialtaan erittäin tiheä.


      • Anonyymi kirjoitti:

        Mitä se huru-ukko höpäjää? Avaruuden havaittu laajeneminen viittaa siihen, että kaukana historiassa avaruus on ollu kaikilta etäisyyksiltään erittäin pieni sekä materialtaan erittäin tiheä.

        Avaruus ei laajene, eikä universumi ala. Se on vaan se älytön ja mahdoton BB teoria kun väittää niin.

        Ja jokaista joka vastustaa, sanotaan huru-ukoksi ja mielisairaaksi jne. Mutta me olemme oikeassa ja BB väärässä. Huono ja mahdoton teoria.

        Universumi ei ole sellainen kuin teoria väittää, vaan sellainen kuin teleskoopeista näkyy: Galakseja ja mustaa.


      • Anonyymi

        Filosofeilla ei ole minkään valtakunnan kompetenssia arvioida alkuräjähdysteoriaa. Toki joku filosofi voi julistaa maapallon olevan litteä, mutta ei sellaisille aivoituksille tarvitse korviaan lotkauttaa eikä varsinkaan tarjota veronmaksajien rahaa.


      • Anonyymi
        Anonyymi kirjoitti:

        Filosofeilla ei ole minkään valtakunnan kompetenssia arvioida alkuräjähdysteoriaa. Toki joku filosofi voi julistaa maapallon olevan litteä, mutta ei sellaisille aivoituksille tarvitse korviaan lotkauttaa eikä varsinkaan tarjota veronmaksajien rahaa.

        Anon.
        "Filosofeilla ei ole minkään valtakunnan kompetenssia arvioida
        alkuräjähdysteoriaa."

        Eivät filosofit rakentele teleskooppeja tutkiakseen alkuräjähdystä,
        he ovat kiinnostuneet kyseisestä käsitteestä, erityisesti
        tieteenfilosofit, jotka eivät kirjoittele tälle palstalle.

        Filosofia on käsitteiden tutkimista tai käsitteellistä tutkimista.
        Joten aloituskommenttisi meni vähän ohitse.

        Mitä tulee litteän maan käsitykseen, sitä ei voi perustella kovinkaan
        vakuuttavasti, joten ainakaan ammattitaitoiset filosofit eivät
        tartu aiheeseen, herjaa voivat tietysti heitellä.

        Heität kevyitä kommentteja filosofeista, mikä on tietysti eräänlaista
        filosofiaa itsekin.
        Mutta miksi oikein ajattelet noin?
        Ja kirjoitat sen filosofiapalstalle!?

        Se ei ole hyvää filosofiaa,


        R.


      • Anonyymi kirjoitti:

        Filosofeilla ei ole minkään valtakunnan kompetenssia arvioida alkuräjähdysteoriaa. Toki joku filosofi voi julistaa maapallon olevan litteä, mutta ei sellaisille aivoituksille tarvitse korviaan lotkauttaa eikä varsinkaan tarjota veronmaksajien rahaa.

        Filosofía miettii universumin ikuisuutta ja äärettömyyttä ja huomaa siinä muutamia loogisia totuuksia, joiden takia universumin, kaikkeuden alkaminen on mahdotonta.

        Sellainen teoria olisi pitänyt heti hylätä eikä pitää sitä ollenkaan tieteellisenä kuin nyt on erehdytty tekemään sata vuotta.


    • Anonyymi

      Erityinen suhteellisuusteoria on väärä nimike: ennemmin tulisi puhua eriskummallisesta teoriasta. Johan sen nyt jo järkikin sanoo ettei latoa pidempi seiväs voi kokonaisena mahtua itseään lyhyemmän ladon sisään! seiväs on sama seiväs vaikka lentäisi kuinka lujaa.

      Toivottavasti Himanka jatkaa ansioikasta työtään terveen järjen palauttamisessa luonnontieteen pariin. Karavaani kulkee ja koirat haukkuu.

      • Anonyymi

        Anon.
        "Erityinen suhteellisuusteoria on väärä nimike:"

        Kari Enqvist sanoo, ettei erikoisessa suhteellisuusteoriassa ole
        mitään erikoista, parempi nimi olisi "suppea suhteellisuusteoria".

        Mitä tulee seiväs-lato paradoksiin, eräs selitys voisi olla hyvä.

        Suhteellisuusteoria on havaitsijakeskeinen, kyse on siitä miten
        eri havaitsijat kokevat asian.

        Seiväs liikkuu lähes valon nopeudella kohti latoa, seiväs on 5 metriä
        pitempi kuin lato.
        On kolme havaitsijaa, yksi lentää seipään mukana, yksi on ladossa
        ja yksi katsoo sivusta mitä tapahtuu.

        Sivusta katseleva näkee seipään lyhenevän, ladossa oleva näkee
        seipään lyhenevän, seipään mukana oleva näkee ladon lyhenevän.

        Kun seiväs menee latoon, sivussa olija näkee seipään mahtuvan
        latoon, ladossa oleva näkee lyhyemmän seipään mahtuvan latoon,
        seipään mukana tuleva näkee ettei seiväs mahdu lyhenevään latoon.

        Sivussa oleva näkee siis koko asetelman, liikkuessaan seiväs on
        lyhyemmän ladon sisällä.
        Tämä koskee tilannetta kun seiväs liikkuu, jos se pysäytettäisiin
        ladossa, sivusta katsoja näkisi seipään olevan 5 metriä ladon ulkopuolella,
        ladossa olija näkisi samoin ja seipään mukana liikkunut näkisi samoin.

        Tämä on otettu erään fyysikon selityksestä, voi olla että muistan
        jotakin väärin, korjatkaa siinä tapauksessa.

        R.


    • Anonyymi

      Taannoinen plusmerkkeihin kohdistunut palstarutto on näköjään syönyt vanhoista viesteistä kaikki plussat pois, vaikka nyt ne toimivat. Eli nyt näkyy vanhoissa viesteissä

      1 1 = 2

      Kun siis niissä alun perin oli
      1+1= 2

    Ketjusta on poistettu 0 sääntöjenvastaista viestiä.

    Luetuimmat keskustelut

    1. Naiset miltä kiihottuminen teissä tuntuu

      Kun miehellä tulee seisokki ja ja sellainen kihmelöinti sinne niin mitä naisessa köy? :)
      Sinkut
      109
      7883
    2. Haistoin ensin tuoksusi

      Käännyin katsomaan oletko se todellakin sinä , otin askeleen taakse ja jähmetyin. Moikattiin naamat peruslukemilla. Tu
      Ikävä
      37
      2882
    3. Olet sä kyllä

      ihme nainen. Mikä on tuo sun viehätysvoiman salaisuus?
      Ikävä
      35
      2472
    4. Teuvo Hakkaraisesta tulee eurovaalien ääniharava

      Persuissa harmitellaan omaa tyhmyyttä
      Maailman menoa
      119
      2185
    5. Hiljaiset hyvästit?

      Vai mikä on :( oonko sanonut jotain vai mitä?
      Ikävä
      17
      1855
    6. Miksi kohtelit minua kuin tyhmää koiraa?

      Rakastin sinua mutta kohtelit huonosti. Tuntuu ala-arvoiselta. Miksi kuvittelin että joku kohtelisi minua reilusti. Hais
      Särkynyt sydän
      10
      1609
    7. Musiikkineuvos Ilkka Lipsanen eli Danny TV:ssä - Blondeja, hittibiisejä, räjäyttävä Danny Show...

      Ilkka Lipsanen eli Danny on viihdyttänyt meitä jo kuusi vuosikymmentä. Musiikkineuvos on myös liikemies, jonka voidaan
      Suomalaiset julkkikset
      38
      1459
    8. Turha mun on yrittää saada yhteyttä

      Oot mikä oot ja se siitä
      Suhteet
      10
      1446
    9. Kyllä poisto toimii

      Esitin illan suussa kysymyksen, joka koska palstalla riehuvaa häirikköä ja tiedustelin, eikö sitä saa julistettua pannaa
      80 plus
      15
      1415
    10. "Joka miekkaan tarttuu, se siihen hukkuu"..

      "Joka miekkaan tarttuu, se siihen hukkuu".. Näin puhui jo aikoinaan Jeesus, kun yksi hänen opetuslapsistaan löi miekalla
      Yhteiskunta
      11
      1364
    Aihe